You are on page 1of 85

SET 2 Block 3: Polity

Exam Title :
& Cur...
Email : misrapulkit@yahoo.in
Contact :

QUESTION 1.
Consider the below statements:

1. Constitution provides for a parliamentary form of government only at the Centre.

2. Parliamentary system and Cabinet system was borrowed from British Constitution.

3. President is just a titular executive or just a nominal executive and not real in Parliamentary
system.

Which of the statements given above is/are correct?

a) 1 and 3 only
b) 2 and 3 only
c) 2 only
d) 3 only
Correct Answer: B
Your Answer: Unanswered
Explanation

Solution (b)

Constitution provides for a parliamentary form of government both at the Centre and in the
states. Hence, statement (1) is wrong.

Art 74 and 75 à deals with the parliamentary system at the Centre

Art 163 and 164 à deals with the parliamentary system in the states

What is Parliamentary system of government?

· Governments have been classified on the basis of relationship of the political executive with
the legislative branch.

· If the executive is answerable and responsible to the legislature it is called Cabinet form
of Government or Parliamentary form of government. In such a system of government, there
exists a very close relationship between the executive and the legislature .

· This form of government is prevalent in Britain, Japan, Canada, India among others.

· The parliamentary government is also known as cabinet government or responsible


government or Westminster model of government .

· India borrowed Parliamentary system and Cabinet system from British Constitution .

In Parliamentary system, the President is the nominal executive (de jure executive or titular exe
cutive ) while the Prime Minister is the real executive (de facto executive). Thus, the President
is head of the State, while the Prime Minister is head of the government.

QUESTION 2.
Consider the below statements about Executive:

IASbaba
Web: http://ilp.iasbaba.com/ Score:
Email: ilp@iasbaba.com 0.00 / 200
Page 1
SET 2 Block 3: Polity
Exam Title :
& Cur...
Email : misrapulkit@yahoo.in
Contact :

1. Executive enjoys the right to get the Legislature dissolved.

2. Ministers are members of both the legislature and the executive.

3. Cabinet headed by PM is the real executive.

Which of the statements given above is/are correct?

a) 2 only
b) 1 and 3 only
c) 2 and 3 only
d) 1, 2 and 3
Correct Answer: D
Your Answer: Unanswered
Explanation

Solution (d)

One of the features or principles of parliamentary government in India is Double Membership


. The ministers are members of both the legislature and the executive . This means that a
person cannot be a minister without being a member of the Parliament. The Constitution
stipulates that a minister who is not a member of the Parliament for a period of six consecutive
months ceases to be a minister.

Another feature or principle of parliamentary government in India is Dissolution of the Lower


House by Executive . This means the lower house of the Parliament (Lok Sabha) can be
dissolved by the President on recommendation of the Prime Minister.

In other words, the prime minister can advise the President to dissolve the Lok Sabha before
the expiry of its term and hold fresh elections. This means that the executive enjoys the right
to get the legislature dissolved in a parliamentary system.

The Cabinet (the real executive) is accountable to the Parliament and stays in office so long
as it enjoys the latter’s confidence. Cabinet is the nucleus of power and Cabinet headed by PM
is the real executive .

QUESTION 3.
Which among the following is/are demerits of Presidential system of government?

1. Conflict between legislature and executive

2. No continuity of policies

3. Prevents despotism

4. Non-responsible government

Choose the correct code from below options:

a) 1 and 2 only
b) 2 and 4 only
c) 1 and 4 only

IASbaba
Web: http://ilp.iasbaba.com/ Score:
Email: ilp@iasbaba.com 0.00 / 200
Page 2
SET 2 Block 3: Polity
Exam Title :
& Cur...
Email : misrapulkit@yahoo.in
Contact :

d) 1, 3 and 4
Correct Answer: C
Your Answer: Unanswered
Explanation

Solution (c)

QUESTION 4.
Constitution has defined the territorial limits of the legislative powers vested in the Centre and
the states. Which of the below statement is not true in regard to it?

a) A state legislature can make laws for the whole or any part of the state.
b) The laws made by a state legislature are not applicable outside the state, except when
there is a sufficient nexus between the state and the object.
c) Parliament can make laws for the whole or any part of the territory of India.
d) Neither parliament nor state legislature can make ‘extra-territorial legislation’.
Correct Answer: D
Your Answer: Unanswered
Explanation

Solution (d)

Territorial Extent of Central and State Legislation

IASbaba
Web: http://ilp.iasbaba.com/ Score:
Email: ilp@iasbaba.com 0.00 / 200
Page 3
SET 2 Block 3: Polity
Exam Title :
& Cur...
Email : misrapulkit@yahoo.in
Contact :

The Constitution defines the territorial limits of the legislative powers vested in the Centre and
the states in the following way:

(i) The Parliament can make laws for the whole or any part of the territory of India. The
territory of India includes the states, the union territories, and any other area for the time being
included in the territory of India.

(ii) A state legislature can make laws for the whole or any part of the state. The laws made by a
state legislature are not applicable outside the state, except when there is a sufficient nexus
between the state and the object.

(iii) The Parliament alone can make ‘extra-territorial legislation’. Thus, the laws of the
Parliament are also applicable to the Indian citizens and their property in any part of the world.

QUESTION 5.
Consider the below statements and choose the correct one/s from the code given below:

1. The subject of ‘Local Government’ is mentioned in the State List under the Eleventh and
Twelfth Schedule of the Constitution.

2. State has to take steps to organize Panchayats and endow them with such powers and
authority as may be necessary to enable them to function as units of self government.

Code:

a) 1 only
b) 2 only
c) Both 1 and 2
d) Neither 1 nor 2
Correct Answer: B
Your Answer: Unanswered
Explanation

Solution (b)

The subject of ‘Local Government’ is mentioned in the State List under the Seventh Schedule of
the Constitution. ( not in Eleventh and Twelfth Schedule, hence statement 1 is wrong)

Article 40 of the Constitution, under Directive Principles (DPSP), says that the state shall take
steps to organize Panchayats and endow them with such powers and authority as may be
necessary to enable them to function as units of self government. Hence, statement 2 is correct.

QUESTION 6.
Parliament can make laws on any matter enumerated in the State List under which of the
following extraordinary circumstances?

1. During a National Emergency

2. When states make a request

IASbaba
Web: http://ilp.iasbaba.com/ Score:
Email: ilp@iasbaba.com 0.00 / 200
Page 4
SET 2 Block 3: Polity
Exam Title :
& Cur...
Email : misrapulkit@yahoo.in
Contact :

3. When Parliament passes a resolution

4. During President’s Rule

Choose the correct code from below options:

a) 1 and 4 only
b) 1, 2 and 4 only
c) 1, 3 and 4 only
d) 1, 2, 3 and 4
Correct Answer: B
Your Answer: Unanswered
Explanation

Solution (b)

Parliament can make laws on any matter enumerated in the State List under the following 5
extraordinary circumstances –

1) When RS passes a resolution (not Parliament, hence option b is the correct answer)

2) During a National Emergency

3) When states make a request

4) To implement International Agreements

5) During President’s Rule

Description:

1) When RS passes a resolution

• If the RS declares that it is necessary in the National Interest that Parliament should make
laws on matter enumerated in the State List
• Such a resolution must be supported by two-thirds of the members present & voting

2) During a National Emergency

• Parliament (makes laws) acquires power to legislate with respect to matters in the State
List while emergency is in operation.
• The laws become inoperative on the expiration of 6months after the emergency ha
s ceased to operate.

3) When states make a request

• When the legislatures of two/more states pass resolution requesting the Parliament
to enact laws on a matter in State List.
• A law so enacted applies only to those states which have passed the resolution. Any other
state may adopt it afterwards by passing a resolution.
• Such a law can be repealed/amended only by the Parliament & not by concerned state
legislatures.

IASbaba
Web: http://ilp.iasbaba.com/ Score:
Email: ilp@iasbaba.com 0.00 / 200
Page 5
SET 2 Block 3: Polity
Exam Title :
& Cur...
Email : misrapulkit@yahoo.in
Contact :
• Ex. Prize Competition Act, 1955; Wildlife (Protection) Act, 1972; Water (Prevention &
Control of Pollution) Act, 1974 etc.

4) To implement International Agreements

• Parliament can make laws for implementing International treaties, agreements or


conventions.
• Ex. UN (Privileges & Immunities) Act, 1947; Geneva Convention Act, 1960; legislation
relating to TRIPS etc.

5) During President’s Rule

• When the President’s rule is imposed on the state, the Parliament can make laws on
matters in State List.
• A law made so by the parliament continues to be operative even after the President’s rule.
• But, such a law can be repealed/altered/re-enacted by the State Legislature.

QUESTION 7.
Consider the below statements about Integrated Judicial System in India and choose the
incorrect statement?

a) Judges of a state High Court are appointed by the President in consultation with CJI and
Governor of the state.
b) They can be transferred and removed only by the President.
c) President can establish a common High Court for 2 or more states.
d) Single system of courts enforces both the Central laws as well as the state laws.
Correct Answer: C
Your Answer: Unanswered
Explanation

Solution (c)

The Constitution of India established an integrated judicial system with the Supreme Court at
the top and the state high courts below it. This single system of courts enforces both the
Central laws as well as the state laws. This is done to eliminate diversities in the remedial
procedure.

The judges of a state high court are appointed by the president in consultation with the Chief
Justice of India and the governor of the state. They can also be transferred and removed by the
president.

The Parliament (not President) can establish a common high court for two or more states.
For example, Maharashtra and Goa or Punjab and Haryana have a common high court. Hence
statement (c) is incorrect statement.

QUESTION 8.
With reference to constitutional amendment bill, which of the statements given below is/are
correct?

1. The constitutional amendment bill must be passed in each House by Absolute Majority.

IASbaba
Web: http://ilp.iasbaba.com/ Score:
Email: ilp@iasbaba.com 0.00 / 200
Page 6
SET 2 Block 3: Polity
Exam Title :
& Cur...
Email : misrapulkit@yahoo.in
Contact :

2. In case of joint sitting, for the purpose of deliberation and passage of constitutional
amendment bill, the bill must be passed by a simple majority.

Choose the appropriate code:

a) 1 only
b) 2 only
c) Both 1 and 2
d) Neither 1 nor 2
Correct Answer: D
Your Answer: Unanswered
Explanation

Solution (d)

The constitutional amendment bill must be passed in each House by a special majority, that is, a
majority (that is, more than 50 per cent) of the total membership of the House and a majority of
two-thirds of the members of the House present and voting. However in 1st statement, it is
provided about Absolute Majority (hence statement 1 is wrong).

There is no provision for holding a joint sitting of both the Houses of Parliament if there is a
deadlock over the passage of a constitutional amendment bill. On the other hand, a provision
for a joint sitting is made in the case of an ordinary bill.

Since, there is no provision of joint sitting, for the purpose of deliberation and passage of
constitutional amendment bill - statement 2 is also wrong.

QUESTION 9.
Consider the following statements in accordance to Duration of panchayats :

1. Every panchayat shall continue for 5 yrs from the date of its 1st meeting.

2. It can be dissolved earlier in accordance with the procedure prescribed by the Constitution.

3. In case, it is dissolved earlier, elections must take place within 6 month of its dissolution.

Which of the statements given above is/are correct?

a) 1 only
b) 1 and 3 only
c) 2 and 3 only
d) 1, 2 and 3
Correct Answer: B
Your Answer: Unanswered
Explanation

Solution (b)

Duration of panchayats

• Every panchayat shall continue for 5 yrs from the date of its 1st meeting.

IASbaba
Web: http://ilp.iasbaba.com/ Score:
Email: ilp@iasbaba.com 0.00 / 200
Page 7
SET 2 Block 3: Polity
Exam Title :
& Cur...
Email : misrapulkit@yahoo.in
Contact :
• It can be dissolved earlier in accordance with the procedure prescribed by the state
legislature (not Constitution, hence statement 2 is wrong).
• In case, it is dissolved earlier, elections must take place within 6 month of its dissolution.

QUESTION 10.
Why our Constitution framers adopted the federal system for India?

1. Due to the large size of the country

2. Due to socio-cultural diversity

3. As it ensures the efficient governance of the country

4. As it reconciles national unity with regional autonomy

Choose the correct code from below options:

a) 1 and 2 only
b) 2 and 4 only
c) 3 and 4 only
d) 1, 2, 3 and 4
Correct Answer: D
Your Answer: Unanswered
Explanation

Solution (d)

The Constitution of India provides for a federal system of government in the country. However,
the term ‘federation’ has no where been used in the Constitution.

In spite of the fact that the Indian Constitution establishes a federal structure, it is indeed very
different than a true federation. The framers of the Constitution have modified the true nature
of Indian federation by incorporating certain unitary features in it.

The framers adopted the federal system due to two main reasons—the large size of the country
and its socio-cultural diversity. They realised that the federal system not only ensures the
efficient governance of the country but also reconciles national unity with regional autonomy.

QUESTION 11.
Consider the below statements with regard to Fundamental Rights and choose the correct
statement:

a) During a national emergency, all Fundamental Rights of Indian citizens can be suspended.
b) Fundamental Rights are available to only citizens of India.
c) Fundamental Rights constitute the bedrock of democratic system in the country.
d) Fundamental Rights makes provision for important social and economic rights.
Correct Answer: C
Your Answer: Unanswered
Explanation

IASbaba
Web: http://ilp.iasbaba.com/ Score:
Email: ilp@iasbaba.com 0.00 / 200
Page 8
SET 2 Block 3: Polity
Exam Title :
& Cur...
Email : misrapulkit@yahoo.in
Contact :

Solution (c)

The Fundamental Rights enshrined in Part III of the Constitution have met with a wide and
varied criticism. The arguments of the critics are:

1. Excessive Limitations

They are subjected to innumerable exceptions, restrictions, qualifications and explanations.


Hence, the critics remarked that the Constitution grants Fundamental Rights with one hand
and takes them away with the other.

2. No Social and Economic Rights

The list is not comprehensive as it mainly consists of political rights. It makes no provision for
important social and economic rights like right to social security, right to work, right to
employment, right to rest and leisure and so on. These rights are made available to the citizens
of advanced democratic countries. Also, the socialistic constitutions of erstwhile USSR or China
provided for such rights.

3. No Clarity

They are stated in a vague, indefinite and ambiguous manner. The various phrases and words
used in the chapter like ‘public order’, ‘minorities’, ‘reasonable restriction’, ‘public interest’ and
so on are not clearly defined. The language used to describe them is very complicated and
beyond the comprehension of the common man. It is alleged that the Constitution was made by
the lawyers for the lawyers. Sir Ivor Jennings called the Constitution of India a ‘paradise for
lawyers’.

4. No Permanency

They are not sacrosanct or immutable as the Parliament can curtail or abolish them, as for
example, the abolition of the fundamental right to property in 1978. Hence, they can become a
play tool in the hands of politicians having majority support in the Parliament. The judicially
innovated ‘doctrine of basic structure’ is the only limitation on the authority of Parliament to
curtail or abolish the fundamental right.

5. Suspension during Emergency

The suspension of their enforcement during the operation of National Emergency (except
Articles 20 and 21) is another blot on the efficacy of these rights. This provision cuts at the
roots of democratic system in the country by placing the rights of the millions of innocent
people in continuous jeopardy. According to the critics, the Fundamental Rights should be
enjoyable in all situations—Emergency or no Emergency.

6. Expensive Remedy

The judiciary has been made responsible for defending and protecting these rights against the
interference of the legislatures and executives. However, the judicial process is too expensive
and hinders the common man from getting his rights enforced through the courts. Hence, the
critics say that the rights benefit mainly the rich section of the Indian Society.

7. Preventive Detention

The critics assert that the provision for preventive detention (Article 22) takes away the spirit
and substance of the chapter on fundamental rights. It confers arbitrary powers on the State
and negates individual liberty. It justifies the criticism that the Constitution of India deals more

IASbaba
Web: http://ilp.iasbaba.com/ Score:
Email: ilp@iasbaba.com 0.00 / 200
Page 9
SET 2 Block 3: Polity
Exam Title :
& Cur...
Email : misrapulkit@yahoo.in
Contact :
with the rights of the State against the individual than with the rights of the individual against
the State. Notably, no democratic country in the world has made preventive detention as an
integral part of their

Constitutions as has been made in India.

8. No Consistent Philosophy

According to some critics, the chapter on fundamental rights is not the product of any
philosophical principle. Sir Ivor Jennings expressed this view when he said that the
Fundamental Rights proclaimed by the Indian Constitution are based on no consistent
philosophy. The critics say that this creates difficulty for the Supreme Court and the high courts
in interpreting the fundamental rights.

SIGNIFICANCE OF FUNDAMENTAL RIGHTS

In spite of the above criticism and shortcomings, the Fundamental Rights are significant in the
following respects:

1. They constitute the bedrock of democratic system in the country.


2. They provide necessary conditions for the material and moral protection of man.
3. They serve as a formidable bulwark of individual liberty.
4. They facilitate the establishment of rule of law in the country.
5. They protect the interests of minorities and weaker sections of society.
6. They strengthen the secular fabric of the Indian State.
7. They check the absoluteness of the authority of the government.
8. They lay down the foundation stone of social equality and social justice.
9. They ensure the dignity and respect of individuals.
10. They facilitate the participation of people in the political and administrative process.

QUESTION 12.
Consider the following statements about Directive Principles of State policy:

1. They form the dominating background to all State action, legislative or executive and also
a guide to the courts in some respects.
2. They enable the opposition to exercise influence and control over the operations of the
government.
3. They serve as a crucial test for the performance of the government.

Which of the above statement(s) is/are true?

a) 1 only
b) 1 and 2 only
c) 3 only
d) All of the above
Correct Answer: D
Your Answer: Unanswered
Explanation

Solution (d)

IASbaba
Web: http://ilp.iasbaba.com/ Score:
Email: ilp@iasbaba.com 0.00 / 200
Page 10
SET 2 Block 3: Polity
Exam Title :
& Cur...
Email : misrapulkit@yahoo.in
Contact :

According to M C Setalvad , the former Attorney General of India, the Directive Principles,
although confer no legal rights and creates no legal remedies, are significant and useful in the
following ways:

1. They are like an ‘Instrument of Instructions’ or general recommendations addressed to all


authorities in the Indian Union. They remind them of the basic principles of the new social and
economic order, which the Constitution aims at building.

2. They have served as useful beacon-lights to the courts. They have helped the courts in
exercising their power of judicial review, that is, the power to determine the constitutional
validity of a law.

Statement 1 à 3. They form the dominating background to all State action, legislative
or executive and also a guide to the courts in some respects.

4. They amplify the Preamble, which solemnly resolves to secure to all citizens of India justice,
liberty, equality and fraternity.

The Directives also play the following roles:

1. They facilitate stability and continuity in domestic and foreign policies in political, economic
and social spheres in spite of the changes of the party in power.

2. They are supplementary to the fundamental rights of the citizens. They are intended to fill in
the vacuum in Part III by providing for social and economic rights.

3. Their implementation creates a favorable atmosphere for the full and proper enjoyment of
the fundamental rights by the citizens. Political democracy, without economic democracy, has
no meaning.

Statement 2 à 4. They enable the opposition to exercise influence and control over the
operations of the government. The Opposition can blame the ruling party on the
ground that its activities are opposed to the Directives .

Statement 3 à 5. They serve as a crucial test for the performance of the government.
The people can examine the policies and programmes of the government in the light of
these constitutional declarations.

6. They serve as common political manifesto. ‘A ruling party, irrespective of its political ideology,
has to recognise the fact that these principles are intended to be its guide, philosopher and
friend in its legislative and executive acts.

QUESTION 13.
In the Constitution of India, development of scientific temper, humanism and the spirit of
inquiry and reform is included in the:

a) Preamble to the Constitution


b) Directive Principles of State Policy
c) Fundamental Duties
d) Fundamental Rights
Correct Answer: C
Your Answer: Unanswered

IASbaba
Web: http://ilp.iasbaba.com/ Score:
Email: ilp@iasbaba.com 0.00 / 200
Page 11
SET 2 Block 3: Polity
Exam Title :
& Cur...
Email : misrapulkit@yahoo.in
Contact :
Explanation

Solution (c)

According to Article 51 A, it shall be the duty of every citizen of India:

(a) to abide by the Constitution and respect its ideals and institutions, the National Flag and the
National Anthem;

(b) to cherish and follow the noble ideals that inspired the national struggle for freedom;

(c) to uphold and protect the sovereignty, unity and integrity of India;

(d) to defend the country and render national service when called upon to do so;

(e) to promote harmony and the spirit of common brotherhood amongst all the people of India
transcending religious, linguistic and regional or sectional diversities and to renounce practices
derogatory to the dignity of women;

(f) to value and preserve the rich heritage of the country’s composite culture;

(g) to protect and improve the natural environment including forests, lakes, rivers and wildlife
and to have compassion for living creatures;

(h) to develop scientific temper, humanism and the spirit of inquiry and reform;

(i) to safeguard public property and to abjure violence;

(j) to strive towards excellence in all spheres of individual and collective activity so that the
nation constantly rises to higher levels of Endeavour and achievement; and

(k) to provide opportunities for education to his child or ward between the age of six and
fourteen years. This duty was added by the 86th Constitutional Amendment Act, 2002.

QUESTION 14.
Which of the following procedures can be amended by Special Majority of Parliament and
Consent of States?

1. Election of the President and its manner


2. Admission or establishment of new states
3. Representation of states in Parliament
4. Use of official language

Select the correct answer using the code given below.

a) 1 and 3 only
b) 2 and 4 only
c) 1, 2 and 4 only
d) All of the above
Correct Answer: A
Your Answer: Unanswered
Explanation

IASbaba
Web: http://ilp.iasbaba.com/ Score:
Email: ilp@iasbaba.com 0.00 / 200
Page 12
SET 2 Block 3: Polity
Exam Title :
& Cur...
Email : misrapulkit@yahoo.in
Contact :

Solution (a)

The procedures that can be amended by Special Majority of Parliament and Consent of
States following provisions are in this way:

1. Election of the President and its manner.

2. Extent of the executive power of the Union and the states.

3. Supreme Court and high courts.

4. Distribution of legislative powers between the Union and the states.

5. Any of the lists in the Seventh Schedule.

6. Representation of states in Parliament.

7. Power of Parliament to amend the Constitution and its procedure (Article 368 itself).

Whereas the following can be done by Simple majority of Parliament

• Admission or establishment of new states


• Use of official language

QUESTION 15.
Under Article 352, the President can declare a national emergency when the security of India
or a part of it is threatened by

1. War

2. External aggression

3. Internal disturbance

Select the correct code:

a) 1 only
b) 2 only
c) 1 and 2 only
d) All of the above
Correct Answer: C
Your Answer: Unanswered
Explanation

Solution (c)

Under Article 352, the President can declare a national emergency when the security of India
or a part of it is threatened by war or external aggression or armed rebellion (but not on the
ground of ‘internal disturbance’)

IASbaba
Web: http://ilp.iasbaba.com/ Score:
Email: ilp@iasbaba.com 0.00 / 200
Page 13
SET 2 Block 3: Polity
Exam Title :
& Cur...
Email : misrapulkit@yahoo.in
Contact :
QUESTION 16.
The legislative power of the Parliament includes making laws

1. on matters not enumerated in the Concurrent List and State List


2. in respect of entries in the State List if two or more State Legislatures consider it
desirable
3. for implementing any treaty, agreement or convention with any country even if it falls in
the State List

Select the correct answer using the code given below.

a) 2 only
b) 1 and 2 only
c) 1 and 3 only
d) 1, 2 and 3
Correct Answer: D
Your Answer: Unanswered
Explanation

Solution (d)

The primary function of Parliament is to make laws for the governance of the country. It has
exclusive power to make laws on the subjects enumerated in the Union List (which at present
has 100 subjects, originally 97 subjects) and on the residuary subjects (that is, subjects not
enumerated in any of the three lists). With regard to Concurrent List (which has at present 52
subjects, originally 47 subjects), the Parliament has overriding powers, that is, the law of
Parliament prevails over the law of the state legislature in case of a conflict between the two.

The Constitution also empowers the Parliament to make laws on the subjects enumerated in the
State List (which at present has 61 subjects, originally 66 subjects) under the following five
abnormal circumstances:

(a) when Rajya Sabha passes a resolution to that effect.

(b) when a proclamation of National Emergency is in operation.

(c) when two or more states make a joint request to the Parliament.

(d) when necessary to give effect to international agreements, treaties and conventions.

(e) when President’s Rule is in operation in the state.

All the ordinances issued by the president (during the recess of the Parliament) must be
approved by the Parliament within six weeks after its reassembly. An ordinance becomes
inoperative if it is not approved by the parliament within that period.

QUESTION 17.
Consider the following statements regarding High Courts of India

1. In India, every state has a High Court in its territory.


2. The Constitution of India provides for a High Court for each state.

IASbaba
Web: http://ilp.iasbaba.com/ Score:
Email: ilp@iasbaba.com 0.00 / 200
Page 14
SET 2 Block 3: Polity
Exam Title :
& Cur...
Email : misrapulkit@yahoo.in
Contact :

Which of the following statement(s) is/are correct?

a) 1 only
b) 2 only
c) Both 1 and 2
d) Neither 1 nor 2
Correct Answer: B
Your Answer: Unanswered
Explanation

Solution (b)

The Constitution of India provides for a high court for each state, but the Seventh
Amendment Act of 1956 authorized the Parliament to establish a common high court
for two or more states or for two or more states and a union territory . The territorial
jurisdiction of a high court is co-terminus with the territory of a state. Similarly, the territorial
jurisdiction of a common high court is co-terminus with the territories of the concerned states
and union territory.

At present, there are 24 high courts in the country. Out of them, three are common high courts.
Delhi is the only union territory that has a high court of its own (since 1966). The other union
territories fall under the jurisdiction of different state high courts. The Parliament can extend
the jurisdiction of a high court to any union territory or exclude the jurisdiction of a high court
from any union territory.

QUESTION 18.
Consider the below statements about Fundamental Rights:

1. They are described as the Magna Carta of India.


2. They are justiciable and are absolute.
3. They are meant for promoting the ideal of political democracy.

Which of the above statement(s) is/are true?

a) 1 only
b) 1 and 2 only
c) 1 and 3 only
d) 1, 2 and 3
Correct Answer: C
Your Answer: Unanswered
Explanation

Solution (c)

Part III of the Constitution - Fundamental Rights - is described as the Magna Carta of India.

The Fundamental Rights are guaranteed by the Constitution to all persons without any
discrimination. They uphold the equality of all individuals, the dignity of the individual, the
larger public interest and unity of the nation.

IASbaba
Web: http://ilp.iasbaba.com/ Score:
Email: ilp@iasbaba.com 0.00 / 200
Page 15
SET 2 Block 3: Polity
Exam Title :
& Cur...
Email : misrapulkit@yahoo.in
Contact :

The Fundamental Rights are meant for promoting the ideal of political democracy. They prevent
the establishment of an authoritarian and despotic rule in the country, and protect the liberties
and freedoms of the people against the invasion by the State. They operate as limitations on the
tyranny of the executive and arbitrary laws of the legislature. In short, they aim at establishing
‘a government of laws and not of men’.

They are justiciable, allowing persons to move the courts for their enforcement, if and when
they are violated.

However, it should be noted that they are not absolute but qualified . The state can impose
reasonable restrictions on them. Hence, statement (2) is wrong.

QUESTION 19.
Which one of the following statements with respect to Local Government in India, is correct?

a) 33% of the seats in local bodies are reserved for women.


b) According to the Indian Constitution, the local government is not an independent tier in the
Federal system
c) The grants-in-aid to Panchayats are from Consolidated fund of India.
d) The state election commission may endow the Panchayats with such powers and authority
as may be necessary to enable them to function as institutions of self-government.
Correct Answer: A
Your Answer: Unanswered
Explanation

Solution (a)

Statement 1 is correct

Statement 2 is wrong the local government is an independent tier in the Federal system

Statement 3 is wrong The grants in aid to Panchayats are from Consolidated fund of State.

Statement 4 is wrong The state legislature may endow the Panchayats with such powers and
authority as may be necessary to enable them to function as institutions of self-government.
Such a scheme may contain provisions for the devolution of powers and responsibilities upon Pa
nchayats at the appropriate level with respect to (a) the preparation of plans for economic
development and social justice; (b) the implementation of schemes for economic development
and social justice as maybe entrusted to them, including those in relation to the 29 matters
listed in the Eleventh Schedule.

QUESTION 20.
Which of the following statements is/are true for Financial Emergency in India?

1. There is no maximum period prescribed for its operation.


2. Repeated parliamentary approval is required for its continuation.
3. Financial Emergency was declared once during financial crisis in 1991.

Select the correct code below

IASbaba
Web: http://ilp.iasbaba.com/ Score:
Email: ilp@iasbaba.com 0.00 / 200
Page 16
SET 2 Block 3: Polity
Exam Title :
& Cur...
Email : misrapulkit@yahoo.in
Contact :

a) 1 only
b) 1 and 2 only
c) 1 and 3 only
d) All of the above
Correct Answer: A
Your Answer: Unanswered
Explanation

Solution (a)

A proclamation declaring financial emergency must be approved by both the Houses of


Parliament within two months from the date of its issue. However, if the proclamation of
Financial Emergency is issued at a time when the Lok Sabha has been dissolved or the
dissolution of the Lok Sabha takes place during the period of two months without approving the
proclamation, then the proclamation survives until 30 days from the first sitting of the Lok Sabh
a after its reconstitution, provided the Rajya Sabha has in the meantime approved it.

Once approved by both the Houses of Parliament, the Financial Emergency continues
indefinitely till it is revoked. This implies two things:

1. There is no maximum period prescribed for its operation; and


2. Repeated parliamentary approval is not required for its continuation.

A resolution approving the proclamation of financial emergency can be passed by either House
of Parliament only by a simple majority, that is, a majority of the members of that house present
and voting.

No Financial Emergency has been declared so far, though there was a financial crisis in 1991.

QUESTION 21.
Constitution has provided for certain provisions with regard to inter-state comity or harmony.
Identify the correct ones from the below:

1. Freedom of inter-state trade, commerce and intercourse.

2. Adjudication of inter-state water disputes.

3. Coordination through inter-state councils.

4. Establishment of zonal councils.

Choose the correct codes from below options:

a) 1, 2 and 3
b) 1, 2 and 4
c) 1, 3 and 4
d) All of the above
Correct Answer: A
Your Answer: Unanswered
Explanation

IASbaba
Web: http://ilp.iasbaba.com/ Score:
Email: ilp@iasbaba.com 0.00 / 200
Page 17
SET 2 Block 3: Polity
Exam Title :
& Cur...
Email : misrapulkit@yahoo.in
Contact :

Solution (a)

The successful functioning of the Indian federal system depends not only on the harmonious
relations and close cooperation between the Centre and the states but also between the states
inter se. Hence, the Constitution makes the following provisions with regard to inter-state
comity:

1. Adjudication of inter-state water disputes.

2. Coordination through inter-state councils.

3. Mutual recognition of public acts, records and judicial proceedings.

4. Freedom of inter-state trade, commerce and intercourse.

In addition, the zonal councils have been established by the Parliament (not provided by
the Constitution, hence option ‘a’ is the correct answer) to promote inter-state
cooperation and coordination.

QUESTION 22.
Among the below given committees, which committee appointed by Government of India
recommended for the establishment of the scheme of 'democratic decentralisation ' for the first
time?

a) Balwant Rai Mehta Committee


b) Ashok Mehta Committee
c) G V K Rao Committee
d) L M Singhvi Committee
Correct Answer: A
Your Answer: Unanswered
Explanation

Solution (a)

Balwant Rai Mehta Committee

· In January 1957, the Government of India appointed a committee (headed by Balwant Rai G
Mehta) to examine the working of the Community Development Programme (1952) and the
National Extension Service (1953) and to suggest measures for their better working.

· The committee submitted its report in November 1957 and recommended the establishment of
the scheme of 'democratic decentralisation '.

The other committees (Ashok Mehta Committee, G V K Rao Committee, L M Singhvi Committee
) deal with reviving and strengthening of the declining Panchayati Raj system in India.

QUESTION 23.
Consider the following statements in regard to Panchayati Raj system in India:

IASbaba
Web: http://ilp.iasbaba.com/ Score:
Email: ilp@iasbaba.com 0.00 / 200
Page 18
SET 2 Block 3: Polity
Exam Title :
& Cur...
Email : misrapulkit@yahoo.in
Contact :

1. 73rd Constitution Amendment Act added 11th schedule to the Constitution of India.

2. The Act gave constitutional status to the panchayati raj institutions.

3. The state governments are under the constitutional obligation to adopt the new Panchayati R
aj system.

Which of the statements given above is/are correct?

a) 1 only
b) 1 and 3 only
c) 2 and 3 only
d) 1, 2 and 3
Correct Answer: D
Your Answer: Unanswered
Explanation

Solution (d)

rd Amendment Act of 1992


Significance of 73

1. The 73 rd Constitution Amendment Act added Part IX (THE PANCHAYATS) to the constitution
and incorporated the concept of Gram Sabha .

It says that there shall be a Gram Sabha in each village which would exercise the powers
and performs functions at the village level as the "State legislature may provide by law"

2. This act also added new articles from 243-A to 243-O (English Alphabet 'O')

3. The CAA also added 11th schedule to the Constitution of India.

4. The act has given a practical shape to Article 40 of the Constitution (DPSP)

It says that, “The State shall take steps to organise village panchayats and endow them with
such powers and authority as may be necessary to enable them to function as units of self-
government.”

5. The act gives a constitutional status to the panchayati raj institutions. It has brought them
under the purview of the justiciable part of the Constitution.

6. The act is a significant landmark in the evolution of grassroot democratic institutions in the
country. It transfers the representative democracy into participatory democracy.

Note:

· The state governments are under the constitutional obligation to adopt the new Panchayati Raj
system.

· The compulsory provisions of the act have to be included in the state laws creating the new Pa
nchayati Raj system.

· The voluntary provisions may be included at the discretion of the states.

IASbaba
Web: http://ilp.iasbaba.com/ Score:
Email: ilp@iasbaba.com 0.00 / 200
Page 19
SET 2 Block 3: Polity
Exam Title :
& Cur...
Email : misrapulkit@yahoo.in
Contact :
QUESTION 24.
Which among the following constitutes the provisions under Right to freedom of religion?

1. Right of minorities to establish and administer cultural and educational institutions.


2. Freedom to manage religious affairs
3. Freedom from payment of taxes for promotion of any religion
4. Protection of language, script and culture of religious minorities

Select the correct answer using the code given below.

a) 2 and 3 only
b) 2 and 4 only
c) 1, 2 and 3 only
d) 2, 3 and 4 only
Correct Answer: A
Your Answer: Unanswered
Explanation

Solution (a)

The below are the provisions that comes under the category of Right to freedom of religion
(Article 25–28)

(a) Freedom of conscience and free profession, practice and propagation of religion (Article 25).

(b) Freedom to manage religious affairs (Article 26).

(c) Freedom from payment of taxes for promotion of any religion (Article 27).

(d) Freedom from attending religious instruction or worship in certain educational institutions
(Article 28).

Cultural and educational rights (Articles 29–30) consists of –

(a) Protection of language, script and culture of minorities (Article 29).

(b) Right of minorities to establish and administer educational institutions (Article 30).

QUESTION 25.
Supreme Court for the first time laid down a new doctrine of the ‘basic structure’ (or ‘basic
features’) of the Constitution in -

a) Shankari Prasad case


b) Golak Nath case
c) Kesavananda Bharati case
d) Minerva Mills case
Correct Answer: C
Your Answer: Unanswered
Explanation

Solution (c)

IASbaba
Web: http://ilp.iasbaba.com/ Score:
Email: ilp@iasbaba.com 0.00 / 200
Page 20
SET 2 Block 3: Polity
Exam Title :
& Cur...
Email : misrapulkit@yahoo.in
Contact :

It was in the Kesavananda Bharati case (1973), the Supreme Court laid down a new doctrine of
the ‘basic structure’ (or ‘basic features’) of the Constitution. It ruled that the constituent power
of Parliament under Article 368 does not enable it to alter the ‘basic structure’ of the
Constitution. This means that the Parliament cannot abridge or take away a Fundamental Right
that forms a part of the ‘basic structure’ of the Constitution.

QUESTION 26.
Consider the below statements in regard to Fundamental Duties:

1. Fundamental Duties are confined to citizens only and do not extend to foreigners.
2. The concept of Fundamental Duties was taken from the constitution of erstwhile USSR.
3. They are non-justiciable but legally enforceable.

Which of the statements given above is/are incorrect?

a) 2 only
b) 3 only
c) 1 and 2 only
d) 1, 2 and 3
Correct Answer: B
Your Answer: Unanswered
Explanation

Solution (b)

The Fundamental Duties in the Indian Constitution are inspired by the Constitution of erstwhile
USSR.

Notably, none of the Constitutions of major democratic countries like USA, Canada, France,
Germany, Australia and so on specifically contain a list of duties of citizens. Japanese
Constitution is, perhaps, the only democratic Constitution in world which contains a list of
duties of citizens.

Following points can be noted with regard to the characteristics of the Fundamental Duties:

1. Some of them are moral duties while others are civic duties.
2. Unlike some of the Fundamental Rights which extend to all persons whether citizens or
foreigners, the Fundamental Duties are confined to citizens only and do not extend to
foreigners.
3. Like the Directive Principles, the fundamental duties are also non-justiciable. The
Constitution does not provide for their direct enforcement by the courts. Moreover, there
is not legal sanction against their violation. However, the Parliament is free to enforce
them by suitable legislation.
4. In other words, Fundamental Duties are not legally enforceable, i.e. without any legal
sanction in case of their violation or non-compliance.

QUESTION 27.

IASbaba
Web: http://ilp.iasbaba.com/ Score:
Email: ilp@iasbaba.com 0.00 / 200
Page 21
SET 2 Block 3: Polity
Exam Title :
& Cur...
Email : misrapulkit@yahoo.in
Contact :

Constitution of India provides for special provision for some states under Part XXI. Which
among the following is/are intention behind them?

1. to meet the aspirations of the people of backward regions of the states


2. to protect the cultural and economic interests of the tribal people of the states
3. to deal with the disturbed law and order condition in some parts of the states
4. to protect the interests of linguistic minorities

Select the correct answer using the code given below.

a) 1 and 2 only
b) 1, 2 and 4 only
c) 1, 2 and 3 only
d) 1, 2, 3 and 4
Correct Answer: C
Your Answer: Unanswered
Explanation

Solution (c)

Articles 371 to 371-J in Part XXI of the constitution contain special provisions for eleven
states viz., Maharashtra, Gujarat, Nagaland, Assam, Manipur, Andhra Pradesh, Sikkim,
Mizoram, Arunachal Pradesh, Telangana and Karnataka.

The intention behind them is –

• to meet the aspirations of the people of backward regions of the states (or)
• to protect the cultural and economic interests of the tribal people of the states (or)
• to deal with the disturbed law and order condition in some parts of the states (or)
• to protect the interests of the local people of the states.

XVII of the Constitution deals with protection of the interests of linguistic minorities.

IASbaba
Web: http://ilp.iasbaba.com/ Score:
Email: ilp@iasbaba.com 0.00 / 200
Page 22
SET 2 Block 3: Polity
Exam Title :
& Cur...
Email : misrapulkit@yahoo.in
Contact :

Image link: http://images.indianexpress.com/2017/09/explained-graph.jpg?w=600

QUESTION 28.
Consider the following statements regarding the administration in scheduled areas:

1. The president is empowered to declare an area to be scheduled area.


2. Each state having scheduled areas has to establish a tribe’s advisory council to advice on
welfare and advancement of the scheduled tribes.
3. The governor is empowered to direct that any particular act of Parliament or the state
legislature does not apply to a scheduled area or apply with specified modifications and
exceptions.

Select the correct code given below

a) 1 and 2 only
b) 1 and 3 only
c) 2 and 3 only
d) 1, 2 and 3

IASbaba
Web: http://ilp.iasbaba.com/ Score:
Email: ilp@iasbaba.com 0.00 / 200
Page 23
SET 2 Block 3: Polity
Exam Title :
& Cur...
Email : misrapulkit@yahoo.in
Contact :
Correct Answer: D
Your Answer: Unanswered
Explanation

Solution (d)

The various features of administration contained in the Fifth Schedule are as follows:

1. Declaration of Scheduled Areas : The president is empowered to declare an area to be a


scheduled area. He can also increase or decrease its area, alter its boundary lines, rescind such
designation or make fresh orders for such redesignation on an area in consultation with the
governor of the state concerned.

2. Executive Power of State and Centre : The executive power of a state extends to the

scheduled areas therein. But the governor has a special responsibility regarding such areas. He
has to submit a report to the president regarding the administration of such areas, annually or
whenever so required by the president. The executive power of the Centre extends to giving
directions to the states regarding the administration of such areas.

3. Tribes Advisory Council: Each state having scheduled areas has to establish a tribes
advisory council to advise on welfare and advancement of the scheduled tribes. It is to consist
of 20 members, three-fourths of whom are to be the representatives of the scheduled tribes in
the state legislative assembly. A similar council can also be established in a state having
scheduled tribes but not scheduled areas therein, if the president so directs.

4. Law applicable to Scheduled Areas: The governor is empowered to direct that any
particular act of Parliament or the state legislature does not apply to a scheduled area or apply
with specified modifications and exceptions. He can also make regulations for the peace and
good government of a scheduled area after consulting the tribes advisory council. Such
regulations may prohibit or restrict the transfer of land by or among members of the scheduled
tribes, regulate the allotment of land to members of the scheduled tribes and regulate the
business of money-lending in relation to the scheduled tribes. Also, a regulation may repeal or
amend any act of Parliament or the state legislature, which is applicable to a scheduled area.
But, all such regulations require the assent of the president.

QUESTION 29.
At the Central level, the subject of ‘urban local government’ is dealt with by which of the
following ministries:

1. Ministry of Urban Development

2. Ministry of Defence

3. Ministry of Home Affairs

Select the correct code given below:

a) 1 and 2 only
b) 2 and 3 only
c) 1 and 3 only
d) All of the above

IASbaba
Web: http://ilp.iasbaba.com/ Score:
Email: ilp@iasbaba.com 0.00 / 200
Page 24
SET 2 Block 3: Polity
Exam Title :
& Cur...
Email : misrapulkit@yahoo.in
Contact :
Correct Answer: D
Your Answer: Unanswered
Explanation

Solution (d)

The system of urban government was constitutionalised through the 74th Constitutional
Amendment Act of 1992. At the Central level, the subject of ‘urban local government’ is dealt
with by the following three ministries:

· Ministry of Urban Development, created as a separate ministry in 1985

· Ministry of Defence in the case of cantonment boards

· Ministry of Home Affairs in the case of Union Territories

QUESTION 30.
Recently , some states are racing to gain special status which confers preferential treatment in
the form of central assistance and tax breaks. Which among the following are the conditions to
categorize states for special status?

1. hilly and difficult terrain


2. low population density or sizable share of tribal population
3. strategic location along borders with neighboring countries
4. economic and infrastructural backwardness
5. non-viable nature of state finances

Select the correct code given below:

a) 1, 2 and 3
b) 1, 2, 3 and 4
c) 1, 3 and 5
d) 1, 2, 3, 4 and 5
Correct Answer: D
Your Answer: Unanswered
Explanation

Solution (d)

The concept of a special category state was first introduced in 1969. The 5th Finance
Commission decided to provide certain disadvantaged states with preferential treatment in the
form of central assistance and tax breaks. Initially three states Assam, Nagaland and Jammu &
Kashmir were granted special status but since then eight more have been included Arunachal
Pradesh, Himachal Pradesh, Manipur, Meghalaya, Mizoram, Sikkim, Tripura and Uttarakhand .

Recently, states like Andhra Pradesh, Orissa, West Bengal, Bihar, Tamil Nadu are racing for the
special status.

Conditions to categorize states for special status :

The special status is given to certain states because of their inherent features; like they might
have a low resource base and cannot mobilize resources for development. Some of the features
required for special status are:

IASbaba
Web: http://ilp.iasbaba.com/ Score:
Email: ilp@iasbaba.com 0.00 / 200
Page 25
SET 2 Block 3: Polity
Exam Title :
& Cur...
Email : misrapulkit@yahoo.in
Contact :

(i) hilly and difficult terrain;

(ii) low population density or sizable share of tribal population;

(iii) strategic location along borders with neighboring countries;

(iv) economic and infrastructural backwardness; and

(v) non-viable nature of state finances.

For further reading: http://www.thehindu.com/news/national/What-is-the-special-


category-status/article14553662.ece

QUESTION 31.
Which of the provisions of the Constitution can be amended only by a special majority of the
Parliament and also with the consent of half of the state legislatures by a simple majority?

1. Supreme Court and high courts

2. Power of Parliament to amend the Constitution and its procedure

3. Elections to Parliament and state legislatures

4. Union territories

Select the correct code given below:

a) 1 and 2
b) 1, 2 and 3
c) 1, 2 and 4
d) 1, 3 and 4
Correct Answer: A
Your Answer: Unanswered
Explanation

Solution (a)

Those provisions of the Constitution which are related to the federal structure of the polity can
be amended by a special majority of the Parliament and also with the consent of half of the state
legislatures by a simple majority.

The following provisions can be amended in this way:

1. Election of the President and its manner.


2. Extent of the executive power of the Union and the states.
3. Supreme Court and high courts.
4. Distribution of legislative powers between the Union and the states.
5. Any of the lists in the Seventh Schedule.
6. Representation of states in Parliament.
7. Power of Parliament to amend the Constitution and its procedure (Article 368 itself).

IASbaba
Web: http://ilp.iasbaba.com/ Score:
Email: ilp@iasbaba.com 0.00 / 200
Page 26
SET 2 Block 3: Polity
Exam Title :
& Cur...
Email : misrapulkit@yahoo.in
Contact :

A number of provisions in the Constitution can be amended by a simple majority of the two
Houses of Parliament outside the scope of Article 368. Among these provisions includes:

• Elections to Parliament and state legislatures


• Union territories

QUESTION 32.
Which one of the following is the correct statement with regard to the Emergency Provisions of
Indian Constitution?

a) The Constitution mentions four types of emergency.


b) Emergency provisions are described from Articles 340 to 356 in the Constitution.
c) Financial Emergency has never been imposed in India.
d) Emergency under Article 356 can be imposed on the failure of Constitutional Machinery at
the Union level
Correct Answer: C
Your Answer: Unanswered
Explanation

Solution (c)

Statement 1 : is wrong. The Constitution Stipulates three types of Emergencies

1. An Emergency due to war, external aggression or armed rebellion (Article 352), popularly
known as National Emergency

2. An Emergency due to the failure of the Constitutional machinery in the states (Article 356), P
resident’s Rule.

3. Financial Emergency due to a threat to the financial stability or credit of India (Article 360)

Statement 2 : is wrong. The Emergency provisions are contained in Part XVIII of the
Constitution, from Articles 352 to 360.

Statement 3 : is right. Never a Financial Emergency had been imposed even during Financial
Crisis of 1991

Statement 4 : is wrong According to Article 356 popularly known as President’s Rule is


imposed due to the failure of the Constitutional machinery in the States and not at union
level.

QUESTION 33.
Consider the below statements about Directive Principles of State Policy:

1. They denote the ideals that the State should keep in mind while formulating policies and
enacting laws.
2. They are the constitutional instructions or recommendations to the State in legislative,
executive and judiciary matters.

IASbaba
Web: http://ilp.iasbaba.com/ Score:
Email: ilp@iasbaba.com 0.00 / 200
Page 27
SET 2 Block 3: Polity
Exam Title :
& Cur...
Email : misrapulkit@yahoo.in
Contact :

Which of the above statement(s) is/are correct?

a) 1 only
b) 2 only
c) Both 1 and 2
d) Neither 1 nor 2
Correct Answer: A
Your Answer: Unanswered
Explanation

Solution (a)

The phrase ‘Directive Principles of State Policy’ denotes the ideals that the State should keep in
mind while formulating policies and enacting laws. These are the constitutional instructions or
recommendations to the State in legislative, executive and administrative matters.

According to Article 36, the term ‘State’ in Part IV has the same meaning as in Part III dealing
with Fundamental Rights. Therefore, it includes the legislative and executive organs (and
not judiciary) of the central and state governments, all local authorities and all other public
authorities in the country. Hence, statement (2) is wrong.

QUESTION 34.
Which of the statements given below are provided by Constitution of India?

1. Constitution itself says that Directive Principles of State Policy shall be the duty of the
State to apply these principles in making laws.
2. The Constitution of India provides for a federal system of government in the country and
the term ‘federation’ instead of 'union' has been used in the Constitution.

Select the correct code given below:

a) 1 only
b) 2 only
c) Both 1 and 2
d) Neither 1 nor 2
Correct Answer: A
Your Answer: Unanswered
Explanation

Solution (a)

The directive principles are meant for promoting the ideal of social and economic democracy.
They seek to establish a ‘welfare state’ in India. However, unlike the Fundamental Rights, the
directives are non-justiciable in nature, that is, they are not enforceable by the courts for their
violation. Yet, the Constitution (Article 37) itself declares that ‘these principles are
fundamental in the governance of the country and it shall be the duty of the state to
apply these principles in making laws’ . Hence, they impose a moral obligation on the state
authorities for their application. But, the real force (sanction) behind them is political, that is,
public opinion.

IASbaba
Web: http://ilp.iasbaba.com/ Score:
Email: ilp@iasbaba.com 0.00 / 200
Page 28
SET 2 Block 3: Polity
Exam Title :
& Cur...
Email : misrapulkit@yahoo.in
Contact :

The Constitution of India provides for a federal system of government in the country. However,
the term ‘federation’ has no where been used in the Constitution . Instead, Article 1 of
the Constitution describes India as a ‘Union of States’.

According to Dr B R Ambedkar , the phrase ‘Union of States’ has been preferred to ‘Federation
of States’ to indicate two things:

(i) the Indian federation is not the result of an agreement among the states like the American
federation; and

(ii) the states have no right to secede from the federation. The federation is union because it is
indestructible.

QUESTION 35.

Which of the following are Compulsory provisions under 73 rd constitutional amendment or


Part IX of the constitution;

1. Reservation of seats (both members and chairpersons) for SCs and STs in panchayats at
all the three levels.
2. Establishment of a State Election Commission for conducting elections to the panchayats .
3. Granting financial powers to the panchayats , that is, authorizing them to levy, collect and
appropriate taxes, duties, tolls and fees.
4. 21 years to be the minimum age for contesting elections to panchayats .

Select the correct answer choosing the right code given below:

a) 1 and 2
b) 1, 2 and 3
c) 2, 3 and 4
d) 1, 2 and 4
Correct Answer: D
Your Answer: Unanswered
Explanation

Solution (d)

COMPULSORY AND VOLUNTARY PROVISIONS (discretionary or optional) provisions (features)


of the 73rd Constitutional Amendment Act (1992) or the Part IX of the Constitution:

A. Compulsory Provisions

1. Organization of Gram Sabha in a village or group of villages.


2. Establishment of panchayats at the village, intermediate and district levels.
3. Direct elections to all seats in panchayats at the village, intermediate and district levels.
4. Indirect elections to the post of chairperson of panchayats at the intermediate and district
levels.
5. 21 years to be the minimum age for contesting elections to panchayats .
6. Reservation of seats (both members and chairpersons) for SCs and STs in pancha
yats at all the three levels.
7. Reservation of one-third seats (both members and chairpersons) for women in panchayats
at all the three levels.

IASbaba
Web: http://ilp.iasbaba.com/ Score:
Email: ilp@iasbaba.com 0.00 / 200
Page 29
SET 2 Block 3: Polity
Exam Title :
& Cur...
Email : misrapulkit@yahoo.in
Contact :
8. Fixing tenure of five years for panchayats at all levels and holding fresh elections within
six months in the event of supersession of any panchayat .
9. Establishment of a State Election Commission for conducting elections to the pan
chayats .
10. Constitution of a State Finance Commission after every five years to review the financial
position of the panchayats .

B. Voluntary Provisions

1. Giving representation to members of the Parliament (both the Houses) and the state
legislature (both the Houses) in the panchayats at different levels falling within their
constituencies.
2. Providing reservation of seats (both members and chairpersons) for backward classes in p
anchayats at any level.
3. Granting powers and authority to the panchayats to enable them to function as
institutions of self-government (in brief, making them autonomous bodies).
4. Devolution of powers and responsibilities upon panchayats to prepare plans for economic
development and social justice; and to perform some or all of the 29 functions listed in the
Eleventh Schedule of the Constitution.
5. Granting financial powers to the panchayats , that is, authorizing them to levy,
collect and appropriate taxes, duties, tolls and fees.

QUESTION 36.
Federal Character of Indian Polity is maintained by which of the following powers of the
Judiciary?

1. Power to adjudicate disputes between the central and state government


2. Power to issue writs
3. Power of judicial review

Select the correct answer using the codes given below:

a) 1 and 2 only
b) 2 and 3 only
c) 1 and 3 only
d) 1, 2 and 3
Correct Answer: C
Your Answer: Unanswered
Explanation

Solution (c)

Power to issue writs is a method of protecting fundamental rights of the people. It has no
relation with the maintenance of federal features. Hence, answer doesn’t include statement (2).

Adjudication of disputes is directly related with federal feature of the constitution. Power of
judicial review in specific cases may preserve the federal feature. Hence answer is option (c).

QUESTION 37.

IASbaba
Web: http://ilp.iasbaba.com/ Score:
Email: ilp@iasbaba.com 0.00 / 200
Page 30
SET 2 Block 3: Polity
Exam Title :
& Cur...
Email : misrapulkit@yahoo.in
Contact :

Consider the following statement regarding the federal system in India:

1. The zonal councils aim at promoting cooperation and coordination between states, union
territories and the Centre.
2. Chairman of Zonal councils is Prime Minister.
3. The Zonal Councils are neither statutory nor constitutional bodies.

Which of the statements given above is/are correct?

a) 1 only
b) 1 and 2 only
c) 2 and 3 only
d) 1 and 3 only
Correct Answer: A
Your Answer: Unanswered
Explanation

Solution (a)

The Zonal Councils are the statutory (and not the constitutional) bodies. They are established
by an Act of the Parliament, that is, States Reorganisation Act of 1956.

The act divided the country into five zones (Northern, Central, Eastern, Western and Southern)
and provided a zonal council for each zone.

Each zonal council consists of the following members:

a) home minister of Central government.

b) chief ministers of all the States in the zone.

c) Two other ministers from each state in the zone.

d) Administrator of each union territory in the zone.

The home minister of Central government is the common chairman of the five zonal councils.
Each chief minister acts as a vice-chairman of the council by rotation, holding office for a period
of one year at a time.

The zonal councils aim at promoting cooperation and coordination between states, union
territories and the Centre. They discuss and make recommendations regarding matters like
economic and social planning, linguistic minorities, border disputes, inter-state transport, and
so on. They are only deliberative and advisory bodies.

QUESTION 38.
Consider the following statements about the District and Regional councils formed under the
Sixth Schedule:

1. The district and regional councils have power to make laws on all the matters for their
territorial jurisdiction subject to the approval of governor.
2. District Council in an autonomous district consists of 30 members and they are elected on
the basis of adult franchise.

IASbaba
Web: http://ilp.iasbaba.com/ Score:
Email: ilp@iasbaba.com 0.00 / 200
Page 31
SET 2 Block 3: Polity
Exam Title :
& Cur...
Email : misrapulkit@yahoo.in
Contact :

Which of the statements given above is/are correct?

a) 1 only
b) 2 only
c) Both 1 and 2
d) Neither 1 nor 2
Correct Answer: D
Your Answer: Unanswered
Explanation

Solution (d)

The various features of administration contained in the Sixth Schedule are as follows:

The tribal areas in the four states of Assam, Meghalaya, Tripura and Mizoram have been
constituted as autonomous districts. Each autonomous district has a district council consisting
of 30 members, of whom four are nominated by the governor and the remaining 26 are
elected on the basis of adult franchise . Hence, statement (2) is wrong.

The district and regional councils administer the areas under their jurisdiction. They can make
laws on certain specified matters (not on all matters) like land, forests, canal water,
shifting cultivation, village administration, inheritance of property, marriage and divorce, social
customs and so on. But all such laws require the assent of the governnor . Hence, statement (1)
is wrong.

The district and regional councils within their territorial jurisdictions can constitute village
councils or courts for trial of suits and cases between the tribes. They hear appeals from them.
The jurisdiction of high court over these suits and cases is specified by the governor.

The district council can establish, construct or manage primary schools, dispensaries, markets,
ferries, fisheries, roads and so on in the district. It can also make regulations for the control of
money lending and trading by non- tribals . But, such regulations require the assent of the
governor.

QUESTION 39.
Power of Judicial Review, provided in the Constitution, is exercised on the basis of which
principle?

a) Due Process of Law


b) Procedure established by Law
c) Arbitration
d) Natural Justice
Correct Answer: B
Your Answer: Unanswered
Explanation

Solution (b)

Judicial Review in India is governed by the principle: ‘Procedure Established by Law’. Under it
the court conducts one test, i.e., whether the law has been made in accordance with the powers

IASbaba
Web: http://ilp.iasbaba.com/ Score:
Email: ilp@iasbaba.com 0.00 / 200
Page 32
SET 2 Block 3: Polity
Exam Title :
& Cur...
Email : misrapulkit@yahoo.in
Contact :
granted by the Constitution to the law-making body and follows the prescribed procedure or
not. It gets rejected when it is held to be violative of procedure established by law.

For full detailed explanation follow this link below –

http://www.yourarticlelibrary.com/essay/judicial-review-in-india-meaning-features-and-
other-details/40369/

QUESTION 40.
Provisions for Grant-in-Aid under Article 243 H for the Panchayats has been made out of which
source?

a) Taxes collected by Union Government


b) Consolidated Fund of State
c) Taxes collected by Panchayats
d) Consolidated Fund of India
Correct Answer: B
Your Answer: Unanswered
Explanation

Solution (b)

The governor of a state shall, after every five years, constitute a finance commission to review
the financial position of the panchayats . It shall make the following recommendations to the
Governor:

1. The principles that should govern:

(a) The distribution between the state and the panchayats of the net proceeds of the taxes,
duties, tolls and fees levied by the state.

(b) The determination of taxes, duties, tolls and fees that may be assigned to the panchayats .

(c) The grants-in-aid to the panchayats from the consolidated fund of the state.

2. The measures needed to improve the financial position of the panchayats .

3. Any other matter referred to it by the governor in the interests of sound finance of the panch
ayats

QUESTION 41.
Goods and Service tax (GST) is a comprehensive indirect tax on manufacture, sale and
consumption of goods and services. What taxes at Center level are incorporated into the GST?

1. Additional Excise Duty


2. Service tax
3. Additional Customs Duty
4. Octroi and Entry Tax

Select the correct code given below

IASbaba
Web: http://ilp.iasbaba.com/ Score:
Email: ilp@iasbaba.com 0.00 / 200
Page 33
SET 2 Block 3: Polity
Exam Title :
& Cur...
Email : misrapulkit@yahoo.in
Contact :

a) 1, 2 and 3
b) 2, 3 and 4
c) 1, 3 and 4
d) All of the above
Correct Answer: A
Your Answer: Unanswered
Explanation

Solution (a)

Taxes at the center and state level incorporated into the


GST are:
At the State Level

• State Value Added Tax/Sales Tax


• Entertainment Tax (Other than the tax levied by the local bodies)
• Octroi and Entry Tax
• Purchase Tax
• Luxury Tax
• Taxes on lottery, betting, and gambling

At the Central level

• Central Excise Duty


• Additional Excise Duty
• Service Tax
• Additional Customs Duty (Countervailing Duty)
• Special Additional Duty of Customs

QUESTION 42.
Consider the following statements:

1. Right to Education is a legal right and not a fundamental right.


2. The State shall provide free and compulsory education to all children of the age of six to
fourteen years in such a manner as the State may determine.

Which of the above statement(s) is/are correct?

a) 1 only
b) 2 only
c) Both 1 and 2
d) Neither 1 nor 2
Correct Answer: B
Your Answer: Unanswered
Explanation

Solution (b)

IASbaba
Web: http://ilp.iasbaba.com/ Score:
Email: ilp@iasbaba.com 0.00 / 200
Page 34
SET 2 Block 3: Polity
Exam Title :
& Cur...
Email : misrapulkit@yahoo.in
Contact :

Right to Education

Article 21 A declares that the State shall provide free and compulsory education to all children
of the age of six to fourteen years in such a manner as the State may determine. Thus, this
provision makes only elementary education a Fundamental Right and not higher or
professional education. This provision was added by the 86th Constitutional Amendment Act
of 2002.

This amendment is a major milestone in the country’s aim to achieve ‘Education for All’. The
government described this step as ‘the dawn of the second revolution in the chapter of citizens’
rights’.

Even before this amendment, the Constitution contained a provision for free and compulsory
education for children under Article 45 in Part IV. However, being a directive principle, it was
not enforceable by the courts. Now, there is scope for judicial intervention in this regard.

This amendment changed the subject matter of Article 45 in directive principles. It now reads
—‘The state shall Endeavour to provide early childhood care and education for all children until
they complete the age of six years.’ It also added a new fundamental duty under Article 51A
that reads —‘It shall be the duty of every citizen of India to provide opportunities for education
to his child or ward between the age of six and fourteen years’.

QUESTION 43.
In what regard the Centre have special relationship with respect to NCT of Delhi as compared
to other states?

1. Centre appoints Lieutenant Governor of NCT of Delhi unilaterally while Governor of a


State is appointed only after consultation with the Chief Minister of the respective State.
2. Chief Minister of NCT of Delhi is appointed by the Lieutenant Governor of NCT of Delhi
while the Chief Minister of a State is appointed by the Governor of that State.

Which of the above statement(s) is/are correct?

a) 1 only
b) 2 only
c) Both 1 and 2
d) Neither 1 nor 2
Correct Answer: D
Your Answer: Unanswered
Explanation

Solution (d)

The 69th Constitutional Amendment Act of 1991 provided a special status to the Union Territory
of Delhi, and redesignated it the National Capital Territory of Delhi and designated the
administrator of Delhi as the lieutenant (lt.) governor. It created a legislative assembly and a
council of ministers for Delhi.

The strength of the assembly is fixed at 70 members, directly elected by the people. The
elections are conducted by the election commission of India. The assembly can make laws on all
the matters of the State List and the Concurrent List except the three matters of the State List,

IASbaba
Web: http://ilp.iasbaba.com/ Score:
Email: ilp@iasbaba.com 0.00 / 200
Page 35
SET 2 Block 3: Polity
Exam Title :
& Cur...
Email : misrapulkit@yahoo.in
Contact :
that is, public order, police and land. But, the laws of Parliament prevail over those made by the
Assembly.

The chief minister is appointed by the President (not by the lt. governor). Hence,
statement (2) is wrong. The other ministers are appointed by the president on the advice of
the chief minister. The ministers hold office during the pleasure of the president. The council of
ministers is collectively responsible to the assembly.

Both Lt. Governor and Governors are appointed unilaterally by the Centre. Hence,
statement (1) is wrong.

QUESTION 44.
What are the remedies to a person whose right to speech and expression (Article 19) is
curtailed by the members of his community?

1. He can move to the Supreme Court under Article 32.


2. He can move to the High Court under Article 226.
3. He only has ordinary legal remedy through lower courts.

Select the correct answer using codes given below:

a) 1 and 2 only
b) 2 and 3 only
c) 3 only
d) 1, 2, and 3
Correct Answer: B
Your Answer: Unanswered
Explanation

Solution (b)

The rights under Article 19 are protected against only the state action and not against the
private individual. When the rights that are available against the State’s action only are violated
by the private individuals, there are no constitutional remedies but only ordinary legal
remedies . Therefore, under the current case the aggrieved can’t move to the Supreme
Court . Though, he can go to the High Court because the writ jurisdiction of the High Court is
wider than that of the Supreme Court, as High Court can issue writs even for the enforcement
of the ordinary rights.

QUESTION 45.
According to Dr. B.R Ambedkar which of the following Articles of the Indian Constitution was to
remain a dead-letter?

a) Article 360
b) Article 356
c) Article 352
d) Article 32
Correct Answer: B

IASbaba
Web: http://ilp.iasbaba.com/ Score:
Email: ilp@iasbaba.com 0.00 / 200
Page 36
SET 2 Block 3: Polity
Exam Title :
& Cur...
Email : misrapulkit@yahoo.in
Contact :
Your Answer: Unanswered
Explanation

Solution (b)

Dr B R Ambedkar , while replying to the critics of this provision in the Constituent Assembly,
hoped that the drastic power conferred by Article 356 would remain a ‘dead-letter’ and would
be used only as a measure of last resort. He observed:

“The intervention of the Centre must be deemed to be barred, because that would be an
invasion on the sovereign authority of the province (state). That is a fundamental proposition
which we must accept by reason of the fact that we have a Federal Constitution. That being so,
if the Centre is to interfere in the administration of provincial affairs, it must be under some
obligation which the Constitution imposes upon the Centre. The proper thing we ought to
expect is that such Articles will never be called into operation and that they would remain a
dead-letter. If at all they are brought into operation, I hope the President who is endowed with
this power will take proper precautions before actually suspending the administration of the
province.”

QUESTION 46.
Which of the following statements is incorrect about the Gram Sabha ?

a) Only adult villagers who have the right to vote can be member of Gram Sabha .
b) Gram Sabha plays a supervisory and monitoring role over Gram Panchayat by approving it
plan of work.
c) The Gram Panchayat has an elected Secretary who is also the Secretary of the Gram Sabha
.
d) For better implementation of some specific tasks, Gram Sabha can form committees.
Correct Answer: C
Your Answer: Unanswered
Explanation

Solution (c)

Only adult villagers who have the right to vote can be member of Gram Sabha . Persons below
18 years of age can't become members.

Gram Sabha plays a supervisory and monitoring role over Gram Panchayat by approving it plan
of work.

Gram Sabha form committees like construction, animal husbandry, etc to carry out some
specific tasks.

The Gram Panchayat has a Secretary who is also the Secretary of the Gram Sabha . This
person is not an elected person but is appointed by the government . The Secretary is
responsible for calling the meeting of the Gram Sabha and Gram Panchayat and keeping a
record of the proceedings.

QUESTION 47.

IASbaba
Web: http://ilp.iasbaba.com/ Score:
Email: ilp@iasbaba.com 0.00 / 200
Page 37
SET 2 Block 3: Polity
Exam Title :
& Cur...
Email : misrapulkit@yahoo.in
Contact :

Which of the below given pairs is/are correct?

1. Secularism : : state doesn't promote any religion as state religion.


2. Parliamentary form of government : : Party with maximum number of votes form
government.
3. Federalism : : Each organ of the state checks on power of others and thus ensures balance
of power.

Select the correct answer using codes given below:

a) 1 only
b) 1 and 2 only
c) 2 and 3 only
d) 1, 2, and 3
Correct Answer: A
Your Answer: Unanswered
Explanation

Solution (a)

Parliamentary form of government is a system of government where executive is the part of


legislature and enjoys the support of majority in legislature. Thus party with support of majority
of members in legislature forms the government. Different nations have different electoral
process.

Secularism is where state doesn't promote any religion as state religion. All religions are
treated equally.

Federalism is a political set up where powers are shared between different tiers of the
government i.e. national, state & local.

Separation of power is a concept where each organ of the state checks on power of others and
thus ensures balance of power.

QUESTION 48.
Which of the following rights are guaranteed under Article 22 of the Constitution?

1. The Right to be informed at the time of arrest of the offence for which the person is being
arrested.
2. The Right to be presented before District Magistrate within 24 hours.
3. The arrestee may be permitted to meet his lawyer during interrogation, though not
throughout the interrogation.

Select the correct answer using codes given below:

a) 1 only
b) 1 and 2 only
c) 2 and 3 only
d) 1, 2, and 3
Correct Answer: B
Your Answer: Unanswered

IASbaba
Web: http://ilp.iasbaba.com/ Score:
Email: ilp@iasbaba.com 0.00 / 200
Page 38
SET 2 Block 3: Polity
Exam Title :
& Cur...
Email : misrapulkit@yahoo.in
Contact :
Explanation

Solution (b)

Article 22 grants protection to persons who are arrested or detained.

The Article 22 has two parts—the first part deals with the cases of ordinary law and the second
part deals with the cases of preventive detention law.

(a) The first part of Article 22 confers the following rights on a person who is arrested or
detained under an ordinary law:

(i) Right to be informed of the grounds of arrest.

(ii) Right to consult and be defended by a legal practitioner.

(iii) Right to be produced before a magistrate within 24 hours, excluding the journey time.

(iv) Right to be released after 24 hours unless the magistrate authorises further detention.

These safeguards are not available to an alien or a person arrested or detained under a
preventive detention law.

The Supreme Court also ruled that the arrest and detention in the first part of Article 22 do not
cover arrest under the orders of a court, civil arrest, arrest on failure to pay the income tax, and
deportation of an alien. They apply only to an act of a criminal or quasi-criminal nature or some
activity prejudicial to public interest.

(b) The second part of Article 22 grants protection to persons who are arrested or detained
under a preventive detention law. This protection is available to both citizens as well as aliens
and includes the following:

(i) The detention of a person cannot exceed three months unless an advisory board reports
sufficient cause for extended detention. The board is to consist of judges of a high court.

(ii) The grounds of detention should be communicated to the detenu . However, the facts
considered to be against the public interest need not be disclosed.

(iii) The detenu should be afforded an opportunity to make a representation against the
detention order.

The clause - The arrestee may be permitted to meet his lawyer during interrogation,
though not throughout the interrogation comes under the Supreme Court guidelines
and not mentioned in the Constitution. Hence, statement (3) is wrong.

QUESTION 49.
Which one of the following statements regarding the levying, collecting and distribution of
Income Tax is correct?

a) The Union levies, collects and distributes the proceeds of income tax between itself and
states.
b) The Union levies, collects and keeps all the proceeds of income tax to itself
c) The Union levies and collects the tax but all the proceeds are distributed among the states

IASbaba
Web: http://ilp.iasbaba.com/ Score:
Email: ilp@iasbaba.com 0.00 / 200
Page 39
SET 2 Block 3: Polity
Exam Title :
& Cur...
Email : misrapulkit@yahoo.in
Contact :

d) Only the surcharge levied on income tax is shared between the Union and the states
Correct Answer: A
Your Answer: Unanswered
Explanation

Solution (a)

India is a Federal state. Power is divided between the Union and the states. The functions have
been classified as exclusively for the centre, exclusively for the state and both for the centre
and the states.

Article 280 of our constitution empowers the President of India to set up a Finance Commission
in India; taxes are divided into six groups:

(I) Taxes Levied, Collected and Retained by the Centre:

These taxes are belonging to the centre exclusively. In other words, no part of the proceeds of
these taxes can be assigned to the states. The following taxes fall under this category:

(i) Corporation Tax (Corporate tax)

(ii) Customs Duties.

(iii) Surcharge on Income Tax.

(iv) Taxes on capital value of assets of individual and companies.

(v) Fees on matters of the Union list.

(II) Taxes Levied by the Centre but Collected and Appropriated by the States:

The following taxes are included in this category:

(i) Stamp duties on bills of exchange, cheques, promissory notes and others.

(ii) Excise duties on medicinal and toilet preparation containing alcohol.

There taxes which form part of the union list are levied by the centre but (a) collected by the
states within which such duties are levied; and (b) collected by the centre when such duties are
levied within any Union Territory.

(III) Taxes Levied and Collected by the Centre but Assigned to the States:

This category includes the following duties and taxes:

(i) Duties on succession to property (other than agricultural land).

(ii) Estate duty on property (other than agricultural land).

(iii) Terminal taxes on goods and passengers carried by railways, sea and airways.

(iv) Taxes on railway fares a freights.

(v) Taxes on transaction in stock exchanges and future markets (other than stamp
duties).

IASbaba
Web: http://ilp.iasbaba.com/ Score:
Email: ilp@iasbaba.com 0.00 / 200
Page 40
SET 2 Block 3: Polity
Exam Title :
& Cur...
Email : misrapulkit@yahoo.in
Contact :

(vi) Taxes on the sale or purchase of newspapers and taxes on advertisements published
in them.

(vii) Taxes on the sale or purchase of goods in course of inter-state trade or commerce
(other than newspaper).

(viii) Taxes on the consignment of goods in the course of inter-state trade or commerce.

The net proceeds of these duties and taxes are assigned to states in accordance with the
principles laid down by the parliament.

(IV) Taxes Levied and Collected by the Centre and Compulsorily Distributed between
the Centre and the States:

Taxes on income (other than agricultural income and corporation tax) shall be levied and
collected by the centre but compulsorily distributed between the centre and the states in such
manner as prescribed by the president on the recommendations of the Finance Commission.
The obligatory sharing of income tax is provided by Article 270 of the Constitution.

(V) Taxes Levied and Collected by the Centre and may be distributed between the
Centre and the States:

Under this category falls the excise duties included in the Union list except those on medicinal
and toilet preparations. These are levied and collected by the centre. The net proceeds of such
duties can be paid to states out of the consolidated Fund of India only if the parliament so
provides.

Further, the principles of distribution shall also be laid down by the parliament. It is to be noted
that sharing of the proceeds, of income tax is obligatory, while that of excise duties is
permissible.

(VI) Taxes Levied and Collected and Retained by States:

The following taxes and duties exclusively belong to states. They are mentioned in the State list.
Every state is entitled to levy, collect and appropriate these taxes. The taxes are

(i) Duty on succession to agricultural land.

(ii) Estate duty on agricultural land.

(iii) Land revenue.

(iv) Tax on agricultural income.

(v) Tax on land and buildings

(vi) Capitation taxes.

(vii) Tax on mineral rights.

(viii) Tax on the consumption or sale of electricity.

(ix) Tax on vehicles.

(x) Tax on the sales and purchase of goods (other than newspaper) for e.g. Sales tax.

(xi) Tolls

IASbaba
Web: http://ilp.iasbaba.com/ Score:
Email: ilp@iasbaba.com 0.00 / 200
Page 41
SET 2 Block 3: Polity
Exam Title :
& Cur...
Email : misrapulkit@yahoo.in
Contact :

(xii) Tax on professions, trades and employment.

QUESTION 50.
The 42nd Amendment Act has added four new Directive Principles to the original list. Identify
the correct ones –

1. To secure opportunities for healthy development of children


2. To promote equal justice and to provide free legal aid to the poor
3. To take steps to secure the participation of workers in the management of industries
4. To protect and improve the environment and to safeguard forests and wild life

Codes:

a) 1 and 2 only
b) 1, 2 and 4 only
c) 1, 3 and 4 only
d) 1, 2, 3 and 4
Correct Answer: D
Your Answer: Unanswered
Explanation

Solution (d)

The 42nd Amendment Act of 1976 added four new Directive Principles to the original list.
They require the State:

1. To secure opportunities for healthy development of children (Article 39).


2. To promote equal justice and to provide free legal aid to the poor (Article 39 A).
3. To take steps to secure the participation of workers in the management of industries
(Article 43 A).
4. To protect and improve the environment and to safeguard forests and wild life (Article 48
A).

The 44th Amendment Act of 1978 added one more Directive Principle, which requires the
State to minimize inequalities in income, status, facilities and opportunities (Article 38).

Again, the 86th Amendment Act of 2002 changed the subject-matter of Article 45 and made
elementary education a fundamental right under Article 21 A. The amended directive requires
the State to provide early childhood care and education for all children until they complete the
age of six years.

The 97th Amendment Act of 2011 added a new Directive Principle relating to co-operative
societies. It requires the state to promote voluntary formation, autonomous functioning,
democratic control and professional management of co-operative societies (Article 43B).

QUESTION 51.
The expression, “Proclamation of Emergency” as used in the Constitution refers to –

IASbaba
Web: http://ilp.iasbaba.com/ Score:
Email: ilp@iasbaba.com 0.00 / 200
Page 42
SET 2 Block 3: Polity
Exam Title :
& Cur...
Email : misrapulkit@yahoo.in
Contact :

a) National Emergency
b) Financial Emergency
c) State Emergency or Constitutional Emergency
d) All of the above
Correct Answer: A
Your Answer: Unanswered
Explanation

Solution (a)

Article 352 Proclamation of Emergency – The Constitution employs the expression


‘proclamation of emergency’ to denote the National Emergency only – i.e. an emergency due to
war, external aggression or armed rebellion.

An Emergency due to the failure of the constitutional machinery in the states (Article 356). This
is popularly known as ‘President’s Rule’. It is also known by two other names—‘State
Emergency’ or ‘constitutional Emergency’. However, the Constitution does not use the word
‘emergency’ for this situation.

Financial Emergency is proclaimed under Article 360 – due to a threat to the financial stability
or credit of India

QUESTION 52.
Consider the following statements in regard to proclamation imposing President’s Rule:

1. The President can proclaim President’s Rule only after receiving a written
recommendation from the Governor of that particular state.
2. It must be approved by both the Houses of Parliament within two months from the date of
its issue.

Which of the above statement(s) is/are correct?

a) 1 only
b) 2 only
c) Both 1 and 2
d) Neither 1 nor 2
Correct Answer: B
Your Answer: Unanswered
Explanation

Solution (b)

According to Article 356, President’s rule can be imposed in a state if a situation has arisen in
which the government of the state cannot be carried on in accordance with the provisions of the
Constitution.

Generally, the governor sends a report in this regard to the Centre and it’s his/her report that
forms the basis for the Union Cabinet’s recommendation to the President for invoking Article
356 to impose President’s rule.

IASbaba
Web: http://ilp.iasbaba.com/ Score:
Email: ilp@iasbaba.com 0.00 / 200
Page 43
SET 2 Block 3: Polity
Exam Title :
& Cur...
Email : misrapulkit@yahoo.in
Contact :

However, the provision also says that the President can take such a decision even “otherwise”
(i.e. even in the absence of governor’s report). But in any case, the President has to be satisfied
that the constitutional machinery has broken down in the state.

An Emergency due to the failure of the constitutional machinery in the states (Article 356). This
is popularly known as ‘President’s Rule’. It is also known by two other names—‘State
Emergency’ or ‘Constitutional Emergency’.

A proclamation imposing President’s Rule must be approved by both the Houses of Parliament
within two months from the date of its issue.

However, if the proclamation of President’s Rule is issued at a time when the Lok Sabha has
been dissolved or the dissolution of the Lok Sabha takes place during the period of two months
without approving the proclamation, then the proclamation survives until 30 days from the first
sitting of the Lok Sabha after its reconstitution, provided the Rajya Sabha approves it in the
mean time.

QUESTION 53.
The Fundamental Rights, enshrined in Part III of the Constitution of India, “are inherent and
cannot be extinguished by any constitutional and statutory provision”. This was made clear in
the case of

a) A.K Goplalan v. State of Madras


b) I.C Golak nath v. State of Punjab
c) State of West Bengal v. Committee for Protection of Democratic Rights, West Bengal
d) Shankari Prasad Singh Deo v. Union of India
Correct Answer: B
Your Answer: Unanswered
Explanation

Solution (b)

The Supreme Court’s judgement in the Golaknath case (1967) ruled that the Parliament cannot
take away or abridge any of the Fundamental Rights, which are ‘sacrosanct’ in nature. In other
words, the Court held that the Fundamental Rights cannot be amended for the implementation
of the Directive Principles.

The Parliament reacted to the Supreme Court’s judgement in the Golaknath Case (1967) by
enacting the 24th Amendment Act (1971) and the 25th Amendment Act (1971). The 24th
Amendment Act declared that the Parliament has the power to abridge or take away any of the
Fundamental Rights by enacting Constitutional Amendment Acts.

QUESTION 54.
Consider the below statements with respect to Schedule V and Schedule VI provisions:

1. Governor is empowered to direct that an act of Parliament does not apply to a scheduled
area, i.e , states belonging to Schedule V

IASbaba
Web: http://ilp.iasbaba.com/ Score:
Email: ilp@iasbaba.com 0.00 / 200
Page 44
SET 2 Block 3: Polity
Exam Title :
& Cur...
Email : misrapulkit@yahoo.in
Contact :
2. President may likewise direct that an act of Parliament does not apply to a tribal area ( aut
onomours district), i.e , states belonging to Schedule VI

Select the correct answer using the code given below:

a) 1 only
b) 2 only
c) Both 1 and 2
d) Neither 1 nor 2
Correct Answer: A
Your Answer: Unanswered
Explanation

Solution (a)

The governor is empowered to direct that an act of Parliament does not apply to a scheduled
area in the state or apply with specified modifications and exceptions.

The Governor of Assam may likewise direct that an act of Parliament does not apply to a tribal
area ( autonomours district) in the state or apply with specified modifications and exceptions.
The President enjoys the same power with respect to tribal areas (autonomous districts) in
Meghalaya, Tripura and Mizoram.

QUESTION 55.
The objectives (or the functions) of the zonal councils does not include which of the following:

a) To help in arresting the growth of acute state-consciousness, regionalism, linguism and


particularistic trends
b) To help in removing the after-effects of separation in some cases so that the process of reor
ganisation , integration and economic advancement may synchronise
c) To secure some kind of political equilibrium between different regions of the country
d) None of the above
Correct Answer: D
Your Answer: Unanswered
Explanation

Solution (d)

The objectives (or the functions) of the zonal councils, in detail, are as follows:

(i) To achieve an emotional integration of the country.

(ii) To help in arresting the growth of acute state-consciousness, regionalism, linguism and
particularistic trends.

(iii) To help in removing the after-effects of separation in some cases so that the process of reor
ganisation , integration and economic advancement may synchronise .

(iv) To enable the Centre and states to cooperate with each other in social and economic
matters and exchange ideas and experience in order to evolve uniform policies.

IASbaba
Web: http://ilp.iasbaba.com/ Score:
Email: ilp@iasbaba.com 0.00 / 200
Page 45
SET 2 Block 3: Polity
Exam Title :
& Cur...
Email : misrapulkit@yahoo.in
Contact :

(v) To cooperate with each other in the successful and speedy execution of major development
projects.

(vi) To secure some kind of political equilibrium between different regions of the country.

QUESTION 56.
‘ Rashtriya Garima Abhiyaan ’ is a National campaign to

a) Rehabilitate the homeless and destitute the persons and provide them with suitable sources
of livelihood.
b) Release the sex workers from their practice and provide them with alternative sources of
livelihood.
c) Eradicate the practice of manual scavenging and rehabilitate the manual scavengers
d) Release the bonded labourers from their bondage and rehabilitate them
Correct Answer: C
Your Answer: Unanswered
Explanation

Solution (c)

Rashtriya Garima Abhiyan (National Campaign for Dignity and Eradication of Manual
Scavenging)

Ministry of Social Justice and Empowerment is responsible for rehabilitation of manual


scavengers and it implements the ‘Self Employment Scheme for Rehabilitation of Manual
Scavengers’ (SRMS). Ministry of Social Justice and Empowerment has associated reputed NGOs
like Safai Karamchari Andolan , Rashtriya Garima Abhiyan , Sulabh International etc. for
identification of manual scavengers and their rehabilitation.

Freedom from Manual Scavenging (Important) - http://pib.nic.in/newsite/mbErel.aspx?


relid=126057

Source: http://www.business-standard.com/article/news-ians/implementation-of-act-for-
rehabilitating-manual-scavengers-sought-117092700007_1.html

QUESTION 57.
Consider the following statements about Ports in India

1. The Major Ports are under the Union List of the Constitution of India

2. Non Major Ports are under the Concurrent List of the Constitution of India

3. Project MAUSAM and SAGAR are customer-oriented initiative of the Government of India to
modernize India's Ports so that port-led development can be augmented and coastlines can be
developed to contribute in India's growth

Select the correct statements

a) 1 and 2

IASbaba
Web: http://ilp.iasbaba.com/ Score:
Email: ilp@iasbaba.com 0.00 / 200
Page 46
SET 2 Block 3: Polity
Exam Title :
& Cur...
Email : misrapulkit@yahoo.in
Contact :

b) 2 and 3
c) 1 and 3
d) All of the above
Correct Answer: A
Your Answer: Unanswered
Explanation

Solution (a)

Union List – Ports declared by or under law made by Parliament or existing law to be major
ports, including their delimitation, and the constitution and powers of port authorities therein.

Concurrent List – Ports other than those declared by or under law made by Parliament or
existing law to be major ports.

Major Ports are under the administrative control of Government of India while the Non-major
ports are governed by the respective Maritime State Governments.

Sagar Mala project is a strategic and customer-oriented initiative of the Government of India to
modernize India's Ports so that port-led development can be augmented and coastlines can be
developed to contribute in India's growth. It looks towards "transforming the existing Ports into
modern world class Ports and integrate the development of the Ports, the Industrial clusters
and hinterland and efficient evacuation systems through road, rail, inland and coastal
waterways resulting in Ports becoming the drivers of economic activity in coastal areas.

Do You Know?

SAGAR (Security and Growth for All in the Region)

· Ensuring peace, stability and prosperity is India in Indian Ocean region.

· Goal is to seek a climate of trust and transparency; respect for international maritime rules
and norms by all countries; sensitivity to each other`s interests; peaceful resolution of maritime
issues; and increase in maritime cooperation.

Project Mausam

· Maritime Routes and Cultural Landscapes across the Indian Ocean.

· Transnational initiative meant to revive its ancient maritime routes and cultural linkages with
countries in the region.

· Project focuses on the natural wind phenomenon, especially monsoon winds used by Indian
sailors in ancient times for maritime trade, that has shaped interactions between countries and
communities connected by the Indian Ocean.

· Project will seek to draw on its ancient linkages with countries in this region as it offers an
alternative which could counter-balance the maritime silk route of China

Think

· Seventh Schedule in the Constitution of India

· Major Ports in India

IASbaba
Web: http://ilp.iasbaba.com/ Score:
Email: ilp@iasbaba.com 0.00 / 200
Page 47
SET 2 Block 3: Polity
Exam Title :
& Cur...
Email : misrapulkit@yahoo.in
Contact :

Source: http://www.livemint.com/Politics/0rVW29S0giuZi8ObLgpQLL/Kandla-Port-
rechristened-as-Deendayal-Port.html

QUESTION 58.
Consider the following statements:

1. The International Solar Alliance was launched at the United Nations Climate Change
Conference in 2015.

2. The Alliance includes all the member countries of the United Nations.

3. It is headquartered in Paris

Which of the statements given above is/are correct?

a) 1 and 2
b) Only 1
c) 1 and 3
d) 1, 2 and 3
Correct Answer: B
Your Answer: Unanswered
Explanation

Solution (b)

ISA

· The International Solar Alliance (ISA) is a treaty based international inter-governmental


alliance of 121 solar resource rich countries lying fully or partially between the Tropics of
Cancer and Capricorn. Any country meeting the above criterion and also being a United
Nations Member can become a Member of ISA.

· The International Solar Alliance (ISA) was launched on 30th November, 2015 as a coalition of
the solar resource rich countries by Mr.Narendra Modi , Hon’ble Prime Minister of India and M
r.François Hollande , Hon’ble President of France in the presence of Mr. Ban Ki Moon,
Secretary General of the United Nations on the first day of the Conference of Parties ( CoP ) 21
to the United Nation’s Framework Convention on Climate Change (UNFCCC) at Paris.

· The Paris Declaration establishing ISA calls ISA an alliance dedicated to the promotion of solar
energy among its member countries. Major objective of ISA is deployment of over 1000 GW of
solar energy and mobilization of more than US $ 1000 billion of investment into the solar
energy. ISA is headquartered at Gurugram , Haryana, in India.

Do You Know?

· The area of Earth located in between the Tropic of Cancer and Tropic of Capricorn is called
the tropical (torrid) zone. The points on the Tropic of Cancer are the northernmost points up to
which the Sun can pass directly overhead. Similarly, the southernmost points are on the Tropic
of Capricorn which follow the same criteria. Location at the north of the Tropic of Cancer shows
the Sun appearing at the south of the zenith. The sunniest countries of the world are on the
African continent, ranging from Somalia- Horn of Africa-, east to Niger, west and north to
Egypt.

IASbaba
Web: http://ilp.iasbaba.com/ Score:
Email: ilp@iasbaba.com 0.00 / 200
Page 48
SET 2 Block 3: Polity
Exam Title :
& Cur...
Email : misrapulkit@yahoo.in
Contact :

· For India, the possible additional benefit from the alliance can be the strengthening ties and
with the major African countries increase goodwill for India among them.

Source: http://pib.nic.in/newsite/PrintRelease.aspx?relid=171122

QUESTION 59.
Consider the following statements

1. ALIMCO is under the aegis of Department of Empowerment of Persons with Disabilities ( DE


PwD )

2. ‘ Divyang Sarathi ’ is a mobile application to provide all relevant information pertaining to


the Department of Empowerment of Persons with Disabilities ( DEPwD )

3. Rashtriya Vayoshri Yojana is scheme for providing Physical Aids and Assisted-living Devices
for Senior citizens belonging to BPL category.

Select the correct statements

a) 1 and 2
b) 2 and 3
c) 1 and 3
d) All of the above
Correct Answer: D
Your Answer: Unanswered
Explanation

Solution (d)

ALIMCO

· It is a Miniratna category - II CPSU under the aegis of DEPwD , Ministry of Social Justice &
Empowerment, Govt . of India.

· ALIMCO provides Aids, Appliances and Components to around 02 Lakhs Disabled Persons ( Pw
Ds ) in the country every year and is the largest Manufacturer of Aids and Appliances for Pwds i
n entire South Asia.

Divyang Sarathi

· This mobile application aims at providing all relevant information pertaining to the
Department of Empowerment of Persons with Disabilities ( DEPwD ), Ministry of Social Justice
and Empowerment, including its various acts, rules, regulations and guidelines, schemes,
information about the various outreach institutions, employment opportunities, and the
disability market in an accessible format.

· The mobile application ‘ Divyang Sarathi ’ is compliant with the principles of UNCRPD for
Universal Access and the provisions of the Rights of Persons with Disabilities Act, 2016. The Act
mandates that all information to be made available in an accessible form. The application is also
an integral part of the ICT component of the Accessible India Campaign.

IASbaba
Web: http://ilp.iasbaba.com/ Score:
Email: ilp@iasbaba.com 0.00 / 200
Page 49
SET 2 Block 3: Polity
Exam Title :
& Cur...
Email : misrapulkit@yahoo.in
Contact :

· The unique features of the mobile application ‘ Divyang Sarathi ’ are its audio notes (text-to-
voice conversion software) embedded in the application which converts the written information
into an audio file as well as the adjustable font size which can be altered as per the user’s
requirement. The mobile application will certainly have a wide outreach as it has been
developed bilingually i.e., the information is available in Hindi as well as English. The
application has been so developed that once downloaded, it can be accessed by any Android
smartphone User who may or may not have access to the internet. The mobile app will be
available for download on Google Playstore .

Rashtriya Vayoshri Yojana

· It is scheme for providing Physical Aids and Assisted-living Devices for Senior citizens
belonging to BPL category. This is a Central Sector Scheme, fully funded by the Central
Government.

· The expenditure for implementation of the scheme will be met from the "Senior Citizens'
Welfare Fund". The Scheme will be implemented through the sole implementing agency -
Artificial Limbs Manufacturing Corporation (ALIMCO), a PSU under the Ministry of Social
Justice and Empowerment.

Source: http://pib.nic.in/newsite/PrintRelease.aspx?relid=171125

QUESTION 60.
Consider the following statements

1. A major component of biogas is Methane and CO2

2. Biogas is produced by aerobic decomposition of biomass by microorganisms

Which of the above statements are correct?

a) 1 only
b) 2 only
c) Both 1 and 2
d) Neither 1 nor 2
Correct Answer: A
Your Answer: Unanswered
Explanation

Solution (a)

Biogas typically refers to a mixture of different gases produced by the breakdown of organic
matter in the absence of oxygen. Biogas can be produced from raw materials such as
agricultural waste, manure, municipal waste, plant material, sewage, green waste or food
waste. Biogas is a renewable energy source.

Biogas can be produced by anaerobic digestion with anaerobic organisms, which digest
material inside a closed system, or fermentation of biodegradable materials.

Biogas is primarily methane (CH4) and carbon dioxide (CO2) and may have small amounts of
hydrogen sulfide (H2S), moisture and siloxanes . The gases methane, hydrogen, and carbon
monoxide (CO) can be combusted or oxidized with oxygen. This energy release allows biogas to

IASbaba
Web: http://ilp.iasbaba.com/ Score:
Email: ilp@iasbaba.com 0.00 / 200
Page 50
SET 2 Block 3: Polity
Exam Title :
& Cur...
Email : misrapulkit@yahoo.in
Contact :
be used as a fuel; it can be used for any heating purpose, such as cooking. It can also be used in
a gas engine to convert the energy in the gas into electricity and heat.

Biogas can be compressed, the same way as natural gas is compressed to CNG, and used to
power motor vehicles. In the UK, for example, biogas is estimated to have the potential to
replace around 17% of vehicle fuel. It qualifies for renewable energy subsidies in some parts of
the world. Biogas can be cleaned and upgraded to natural gas standards, when it becomes bio-
methane. Biogas is considered to be a renewable resource because its production-and-use cycle
is continuous, and it generates no net carbon dioxide. Organic material grows, is converted and
used and then regrows in a continually repeating cycle. From a carbon perspective, as much
carbon dioxide is absorbed from the atmosphere in the growth of the primary bio-resource as is
released when the material is ultimately converted to energy.

Anaerobic digestion is a collection of processes by which microorganisms break down


biodegradable material in the absence of oxygen. The process is used for industrial or domestic
purposes to manage waste or to produce fuels. Much of the fermentation used industrially to
produce food and drink products, as well as home fermentation, uses anaerobic digestion.

Think

· Ethanol Blending

Source: http://www.business-standard.com/article/government-press-release/speech-
by-the-president-of-india-shri-ram-nath-kovind-on-117092600866_1.html

QUESTION 61.
Consider the following statements about Scorpene -class submarine

1. It features diesel propulsion and additional air-independent propulsion (AIP)

2. INS Khanderi is India’s first Scorpene -class submarine

3. INS Kalvari is a diesel-electric attack submarine

Select the correct statements

a) 1 and 2
b) 2 and 3
c) 1 and 3
d) All of the above
Correct Answer: C
Your Answer: Unanswered
Explanation

Solution (c)

The Scorpène -class submarines are a class of diesel-electric attack submarines jointly
developed by the French Direction des Constructions Navales (DCN) and the Spanish company

IASbaba
Web: http://ilp.iasbaba.com/ Score:
Email: ilp@iasbaba.com 0.00 / 200
Page 51
SET 2 Block 3: Polity
Exam Title :
& Cur...
Email : misrapulkit@yahoo.in
Contact :
Navantia , and now by DCNS. It features diesel propulsion and an additional air-independent
propulsion (AIP).

INS Kalvari

• Kalvari which is named after the dreaded Tiger Shark is equipped with state of the art
technology, including superior stealth features such as “advanced acoustic silencing
techniques”, low radiated noise and the ability to launch a crippling attack on the enemy
using precision guided weapons.
• It was built at MDL in collaboration with France’s DCNS as part of Project 75 of Indian
Navy.
• It is the first of the Scorpene class submarines.
• It is the stealthiest diesel-electric attack submarine

INS Khanderi

• INS Khanderi , second of the Scorpene class submarine, was launched in Mumbai.
• INS Khanderi is designed to operate in all theatres, including the tropics. All means and
communications are provided to ensure interoperability with other components of a Naval
Task Force.
• It includes superior stealth and the ability to launch a crippling attack on the enemy using
precision guided weapons. The stealth features are expected to give the submarine an
invulnerability , unmatched by many submarines.
• INS Khanderi can undertake diverse types of missions including anti-surface warfare,
anti-submarine warfare, intelligence gathering, mine laying, area surveillance and others.
The features of this submarine also include the ability to launch attacks with torpedoes, as
well as tube-launched anti-ship missiles, whilst underwater or on surface.
• Khanderi is the second of the six submarines being built at MDL in collaboration with M/s
DCNS of France, as part of Project 75 of Indian Navy. The first one is Kalvari which is
currently completing sea trials.
• INS Khanderi is named after the Island fort of Maratha forces, which played a vital role in
ensuring their supremacy at sea in the late 17th century.

Source: http://www.financialexpress.com/industry/make-in-india-scorpene-submarine-
ins-kalvari-ready-for-indian-navy-set-to-be-commissioned-by-pm-modi-soon/869320/

QUESTION 62.
Consider the following statements

1. He played a leading role in the passage of the Widow Remarriage Act of 1856

2. He was appointed as first principal of Sanskrit College

3. He was revered as ‘the father of modern Bengali prose’

Identify the personality

a) Ishwar Chandra Vidyasagar


b) Ram Mohan Roy
c) Dayananda Saraswati
d) Keshub Chandra Sen
Correct Answer: A

IASbaba
Web: http://ilp.iasbaba.com/ Score:
Email: ilp@iasbaba.com 0.00 / 200
Page 52
SET 2 Block 3: Polity
Exam Title :
& Cur...
Email : misrapulkit@yahoo.in
Contact :
Your Answer: Unanswered
Explanation

Solution (a)

Widow Remarriage Act 1856

· Vidyasagar championed the uplift of the status of women in India, particularly in his native
Bengal. Unlike some other reformers who sought to set up alternative societies or systems, he
sought to transform orthodox Hindu society from within.[6]

· With support from people like Akshay Kumar Dutta , Vidyasagar introduced the practice of
widow remarriages to mainstream Hindu society. The prevailing custom of Kulin Brahmin
polygamy allowed elderly men — sometimes on their deathbeds — to marry teenage or
prepubescent girls, supposedly to spare their parents the shame of having an unmarried girl
attain puberty in their house. After such marriages, these girls would usually be left behind in
their parental homes, where they might be subjected to orthodox rituals, especially if they were
subsequently widowed. These included a semi-starvation, hard domestic labour , and close
restriction on their freedom to leave the house or be seen by strangers.

· Unable to tolerate the ill treatment, many of these girls would run away and turn to
prostitution to support themselves . Ironically, the economic prosperity and lavish lifestyles of
the city made it possible for many of them to have successful careers once they stepped out of
the sanction of society and into the demi-monde. In 1853 it was estimated that Calcutta had a
population of 12,718 prostitutes and public women. Many widows had to shave their heads and
don white saris, supposedly to discourage attention from men. They led a deplorable life, somet
hing Vidyasagar thought was unfair and sought to change.

Sanskrit College

· The institution rose to prominence during the principalship of Ishwar Chandra Vidyasagar in
1851, who admitted students from other than the brahmin caste. In particular the tol or
traditional Indian training school model was incorporated as a department in the 1870s.

· From 1824 until 1851 the College did not have the post of Principal but was headed by a
Secretary. From 1851 the College was headed by a Principal.

He was revered as ‘the father of modern Bengali prose’ by Rabindranath Tagore.

He reconstructed the Bengali alphabet and simplified Bengali typography into an alphabet
(actually abugida ) of twelve vowels and forty consonants, eliminating the Sanskrit phonemes 09 E0

(ṝ), 09 8C (ḷ), E1 (ḹ), and AC ( vô ) and a few punctuation marks, while adding three new letters, A1BC ( ṛô ),
09 09 0909

A2BC ( ṛhô ), and AFBC ( yô ), to reflect contemporary pronunciation. He contributed significantly to


0909 0909

Bengali and Sanskrit literature, with one of his works, Borno Porichoy ("Character
Identification"), is being considered a classic literary work.

Source: Ishwar Chandra Vidyasagar was born on this day September 26

QUESTION 63.
The Kurds are scattered mainly over which of the following countries?

a) Iraq
b) Iran

IASbaba
Web: http://ilp.iasbaba.com/ Score:
Email: ilp@iasbaba.com 0.00 / 200
Page 53
SET 2 Block 3: Polity
Exam Title :
& Cur...
Email : misrapulkit@yahoo.in
Contact :

c) Turkey
d) All of the above
Correct Answer: D
Your Answer: Unanswered
Explanation

Solution (d)

Contemporary use of the term refers to the following areas: southeastern Turkey (Northern
Kurdistan), northern Syria ( Rojava or Western Kurdistan), northern Iraq (Southern Kurdistan),
and northwestern Iran (Eastern Kurdistan).

Read More - http://www.bbc.com/news/world-middle-east-29702440

Source: http://www.thehindu.com/news/international/why-kurds-want-a-separate-
country/article19752257.ece

QUESTION 64.
Consider the following statements with respect to Organisation of Islamic Cooperation (OIC)

1. India and Pakistan are members of the OIC

2. OIC regard parts of Kashmir as “occupied by India”

3. It is headquartered at Abu Dhabi, UAE

Which of the following statements is/are correct?

a) Only 2
b) 1 and 2
c) 1 and 3
d) None of the above
Correct Answer: A
Your Answer: Unanswered
Explanation

Solution (a)

India is not a member of OIC but Pakistan is.

India has been blocked by Pakistan from joining the OIC over Kashmir issue. OIC regard parts
of Kashmir as “occupied by India”. The Organisation of Islamic Cooperation (OIC) has voiced its
concern over alleged excesses of the Indian military and the paramilitary forces in Kashmir.

It is headquartered at Jeddah, Saudi Arabia.

Source: http://www.livemint.com/Politics/XMySUL88Ak7xObAcDsnX3O/Kashmir-issue-
should-be-resolved-bilaterally-by-India-Pakis.html

QUESTION 65.

IASbaba
Web: http://ilp.iasbaba.com/ Score:
Email: ilp@iasbaba.com 0.00 / 200
Page 54
SET 2 Block 3: Polity
Exam Title :
& Cur...
Email : misrapulkit@yahoo.in
Contact :

Consider the following statements about Bakhshali manuscript

1. The Bakhshali scroll is recognised as the oldest Indian mathematical text

2. It contains the earliest known Indian use of a zero symbol

Select the correct statements

a) 1 Only
b) 2 Only
c) Both 1 and 2
d) Neither 1 nor 2
Correct Answer: C
Your Answer: Unanswered
Explanation

Solution (c)

The Bakhshali manuscript is a mathematical text written on birch bark that was found in 1881
in the British-ruled village of Bakhshali (near Mardan in present-day Pakistan).

It is notable for being "the oldest extant manuscript in Indian mathematics", with portions dated
to AD 224–383. It contains the earliest known Indian use of a zero symbol.

IASbaba
Web: http://ilp.iasbaba.com/ Score:
Email: ilp@iasbaba.com 0.00 / 200
Page 55
SET 2 Block 3: Polity
Exam Title :
& Cur...
Email : misrapulkit@yahoo.in
Contact :

Source: http://www.livemint.com/Science/MSxVoPsCyzUQcvUMh10hDN/1800yearold-
black-dot-in-Bakhshali-manuscript-is-first-z.html

QUESTION 66.
Consider the following statements about Comptroller and Auditor General (CAG) of India

1. The CAG is mentioned in the Constitution of India

2. The CAG is appointed by the Prime Minister of India

3. The CAG can audit expenditure from the Consolidated Fund of India

Select the correct statements

a) 1 and 2
b) 2 and 3
c) 1 and 3
d) All of the above
Correct Answer: C
Your Answer: Unanswered
Explanation

Solution (c)

The Comptroller and Auditor General (CAG) of India is an authority, established by the
Constitution of India under Chapter V, who audits all receipts and expenditure of the
Government of India and the state governments, including those of bodies and authorities
substantially financed by the government. The CAG is also the external auditor of government-
owned companies. The reports of the CAG are taken into consideration by the Public Accounts
Committees, which are special committees in the Parliament of India and the state legislatures.
The CAG is also the head of the Indian Audit and Accounts Service, which has over 58,000
employees across the country. The CAG is mentioned in the Constitution of India under Article
148 – 151.

The CAG is appointed by the President of India by a warrant under his hand and seal. He holds
office for a period of six years or upto the age of 65 years, whichever is earlier

The powers of the CAG, regarding audit, are provided for in the Comptroller and Auditor
General of India (Duties, Powers and Conditions of Service) Act, 1971. According to this act, the
CAG can audit:

· All receipts and expenditure from the Consolidated Fund of India and of the states and union
territories.

· All transactions relating to the Contingency Funds and Public Accounts.

· All trading, manufacturing, profit and loss accounts and balance sheets and other subsidiary
accounts kept in any department.

· All stores and stock of all government offices or departments.

· Accounts of all government companies set up under the Indian Companies Act, 1956.

IASbaba
Web: http://ilp.iasbaba.com/ Score:
Email: ilp@iasbaba.com 0.00 / 200
Page 56
SET 2 Block 3: Polity
Exam Title :
& Cur...
Email : misrapulkit@yahoo.in
Contact :

· Accounts of all central government corporations whose Acts provide for audit by the CAG.

· Accounts of all authorities and bodies substantially funded from the Consolidated Fund.

Source: http://www.thehindubusinessline.com/news/rajiv-mehrishi-takes-oath-as-the-
new-comptroller-and-auditor-general/article9871511.ece

QUESTION 67.
Consider the following statements about Mangalyaan launched by ISRO

1. It is also called the Mars Orbiter Mission

2. It made India the second country to have a spacecraft orbit the Mars after USA

3. It made India the only country to be successful in making its spacecraft orbit the Mars in its
very first attempt

Which of the statements given above is/are correct?

a) 1 only
b) 2 and 3 only
c) 1 and 3 only
d) 1, 2 and 3
Correct Answer: C
Your Answer: Unanswered
Explanation

Solution (c)

Mars Orbiter Mission is India's first interplanetary mission to planet Mars with an orbiter craft
designed to orbit Mars in an elliptical orbit.

India is the only country to be successful in making its spacecraft orbit Mars in its very first
attempt.

India is the first Asian country and ISRO is the fourth space agency to send a satellite to Mars.

Launch Vehicle: PSLV-C25

Read More - http://www.isro.gov.in/Spacecraft/mars-orbiter-mission-spacecraft

Source: http://www.thehindu.com/opinion/op-ed/on-mangalyaans-first-anniversary-
understanding-the-gains-from-indias-foray-to-mars-and-the-moon/article7685529.ece

QUESTION 68.
The repealed acts TADA and POTA were concerned with

a) Terrorism
b) Unlawful Activities

IASbaba
Web: http://ilp.iasbaba.com/ Score:
Email: ilp@iasbaba.com 0.00 / 200
Page 57
SET 2 Block 3: Polity
Exam Title :
& Cur...
Email : misrapulkit@yahoo.in
Contact :

c) Salary, Allowances and Pension of Members of Parliament


d) The Leaders and Chief Whips of Recognised Parties and Groups in Parliament
Correct Answer: A
Your Answer: Unanswered
Explanation

Solution (a)

Terrorist and Disruptive Activities (Prevention) Act, commonly known as TADA, was an Indian
anti-terrorism law which was in force between 1985 and 1995 (modified in 1987) under the
background of the Punjab insurgency and was applied to whole of India. It came into effect on
23 May 1985. It was renewed in 1989, 1991 and 1993 before being allowed to lapse in 1995
due to increasing unpopularity after widespread allegations of abuse. It was the first anti-
terrorism law legislated by the government to define and counter terrorist activities.

The Prevention of Terrorism Act, 2002 (POTA) was an Act passed by the Parliament of India in
2002, with the objective of strengthening anti-terrorism operations. The Act was enacted due to
several terrorist attacks that were being carried out in India and especially in response to the
attack on the Parliament. The Act replaced the Prevention of Terrorism Ordinance (POTO) of
2001 and the Terrorist and Disruptive Activities (Prevention) Act (TADA) (1985–95), and was
supported by the governing National Democratic Alliance. The Act was repealed in 2004 by the
United Progressive Alliance coalition.

Source: http://indianexpress.com/article/india/2002-03-mumbai-multiple-blasts-
supreme-court-dismisses-convicts-plea-4851263/

QUESTION 69.
Which of the following statements is/are correct?

1. India's Travel and Tourism sector was also the fastest growing amongst the G20 countries
according to the World Travel and Tourism Council (WTTC)

2. Travel and Tourism Competitiveness Index (TTCI) released by the World Travel and Tourism
Council (WTTC)

Select the correct statements

a) 1 Only
b) 2 Only
c) Both 1 and 2
d) Neither 1 nor 2
Correct Answer: A
Your Answer: Unanswered
Explanation

Solution (a)

The World Travel & Tourism Council (WTTC) is a forum for the travel and tourism industry. It is
made up of members from the global business community and works with governments to raise
awareness about the travel and tourism industry. It is known for being the only forum to
represent the private sector in all parts of the industry worldwide. Its activities include

IASbaba
Web: http://ilp.iasbaba.com/ Score:
Email: ilp@iasbaba.com 0.00 / 200
Page 58
SET 2 Block 3: Polity
Exam Title :
& Cur...
Email : misrapulkit@yahoo.in
Contact :
research on the economic and social impact of the industry and its organisation of global and
regional summits focused on issues and developments relevant to the industry.

India's travel and tourism sector ranks 7th in the world in terms of its total contribution to the
country's GDP, according to a new report of the World Travel and Tourism Council (WTTC)

According to data, the travel and tourism sector generated Rs 14.1 trillion (USD208.9 billion) in
2016, which is world's 7th largest in terms of absolute size; the sum is equivalent to 9.6% of
India's GDP.

Additionally, the sector created 40.3 million jobs in 2016, which ranks India 2nd in the world in
terms of total employment generated. The sector accounts for 9.3% of the country's total jobs.

India's Travel and Tourism sector was also the fastest growing amongst the G20 countries,
growing by 8.5% in 2016. A further 6.7% growth is forecast for 2017.

WTTC said India's figures are predominantly generated by domestic travel, which accounts for
88% of the sector's contribution to GDP in 2016.

Travel and Tourism Competitiveness Index (TTCI) released by the World Economic Forum
(WEF)

Source: http://economictimes.indiatimes.com/industry/services/travel/indias-is-the-
worlds-7th-largest-tourism-economy-in-terms-of-gdp-says-wttc/
articleshow/58011112.cms

QUESTION 70.
Consider the following about Sansad Adarsh Gram Yojana (SAGY)

1. It is a social security and welfare programme to provide support to aged persons, widows,
disabled persons and bereaved families on death of primary bread winner, belonging to below
poverty line households.

2. Annapurna Scheme is a component of SAGY

Select the correct statements

a) 1 Only
b) 2 Only
c) Both 1 and 2
d) Neither 1 nor 2
Correct Answer: D
Your Answer: Unanswered
Explanation

Solution (d)

Sansad Adarsh Gram Yojana (SAGY) is a village development project launched by Government
of India in October 2014, under which each Member of Parliament will take the responsibility of
developing physical and institutional infrastructure in three villages by 2019. The Saansad Adar
sh Gram Yojana (SAANJHI) was launched on on 11th October, 2014.

IASbaba
Web: http://ilp.iasbaba.com/ Score:
Email: ilp@iasbaba.com 0.00 / 200
Page 59
SET 2 Block 3: Polity
Exam Title :
& Cur...
Email : misrapulkit@yahoo.in
Contact :

The goal is to develop three Adarsh Grams by March 2019, of which one would be achieved by
2016. Thereafter, five such Adarsh Grams (one per year) will be selected and developed by
2024.

In order to achieve these objectives, SAGY would be guided by the following approach:

· Leveraging the leadership, capacity, commitment and energy of the Members of Parliament
(MP) to develop model Gram Panchayats

· Engaging with and mobilizing the community for participatory local level development.

· Converging different government programmes and private and voluntary initiatives to achieve
comprehensive development in tune with people’s aspirations and local potential.

· Building partnerships with voluntary organisations , co-operatives and academic and research
institutions.

· Focusing on outcomes and sustainability.

The MP would be free to identify a suitable Gram Panchayat for being developed as Adarsh Gra
m, other than his/her own village or that of his/her spouse.

The MP will identify one Gram Panchayat to be taken up immediately, and two others to be
taken up a little later. Lok Sabha MP has to choose a Gram Panchayat from within his/her
constituency and Rajya Sabha MP a Gram Panchayat from the rural area of a district of his/her
choice in the State from which he/she is elected. Nominated MPs may choose a Gram Panchayat
from the rural area of any district in the country. In the case of urban constituencies, (where
there are no Gram Panchayats ), the MP will identify a Gram Panchayat from a nearby rural
constituency.

IASbaba
Web: http://ilp.iasbaba.com/ Score:
Email: ilp@iasbaba.com 0.00 / 200
Page 60
SET 2 Block 3: Polity
Exam Title :
& Cur...
Email : misrapulkit@yahoo.in
Contact :

The Gram Panchayats once selected by members of Parliament (whose tenures have ended on
account of resignation or otherwise) would be continued as such under SAGY irrespective of
whether activities have already been initiated in the GP under SAGY or not. The newly elected
MPs will have the option to select the GP of their choice and two more subsequently by 2019.

Primarily, the goal is to develop three Adarsh Grams by March 2019, of which one would be
achieved by 2016. Thereafter, five such Adarsh Grams (one per year) will be selected and
developed by 2024.

Source: http://www.business-standard.com/article/government-press-release/
celebrating-science-for-new-india-dr-harshvardhan-addresses-a-curtain-
raiser-117092901047_1.html

QUESTION 71.
Consider the following statements about National Remote Sensing Centre (NRSC)

1. It is a part of Indian Space Research Organization (ISRO)

2. It is the focal point for distribution of remote sensing satellite data products in India and its n
eighbouring countries

Select the correct statements

a) 1 Only
b) 2 Only
c) Both 1 and 2
d) Neither 1 nor 2
Correct Answer: C
Your Answer: Unanswered
Explanation

Solution (c)

National Remote Sensing Centre (NRSC ), is a full-fledged centres of ISRO. NRSC was
functioning as an autonomous body called National Remote Sensing Agency (NRSA) under
Department of Space (DOS) till August, 2008. The Centre is responsible for remote sensing
satellite data acquisition and processing, data dissemination, aerial remote sensing and
decision support for disaster management.

NRSA was transferred from DST to new created Department of Space in 1980 as an
autonomous centre . Indian Photo-interpretation Institute was renamed as Indian Institute of
Remote Sensing in 1983. On 1st November 1995, the UN affiliated Centre for Space Science
and Technology Education in Asia and the Pacific (CSSTE-AP) was created and hosted at Indian
Institute of Remote Sensing (IIRS), Dehradun. On September 1, 2008 NRSA was converted from
a autonomous organisation to a fully Government organisation under ISRO and renamed
National Remote Sensing Centre (NRSC).

National Remote Sensing Centre (NRSC) is the focal point for distribution of remote sensing
satellite data products in India and its neighboring countries. NRSC has an earth station at Sha
dnagar , about 55Km from Hyderabad, to receive data.

IASbaba
Web: http://ilp.iasbaba.com/ Score:
Email: ilp@iasbaba.com 0.00 / 200
Page 61
SET 2 Block 3: Polity
Exam Title :
& Cur...
Email : misrapulkit@yahoo.in
Contact :

Think

· Bhoosampada

· Bhuvan Portal

Source: http://www.hindustantimes.com/india-news/use-geo-spatial-tech-for-cleaning-
ganga-union-minister-to-officials/story-ounp97s3eJg2xRRbPC3cxH.html

QUESTION 72.
Consider the following statements about National Mission for Clean Ganga (NMCG)

1. It acts as implementation arm of National Council for Rejuvenation, Protection and


Management of River Ganga

2. National Ganga Council is under the chairmanship of Prime Minister of India

Select the correct statements

a) 1 Only
b) 2 Only
c) Both 1 and 2
d) Neither 1 nor 2
Correct Answer: C
Your Answer: Unanswered
Explanation

Solution (c)

National Mission for Clean Ganga (NMCG) was registered as a society on 12th August 2011
under the Societies Registration Act 1860.It acted as implementation arm of National Ganga
River Basin Authority (NGRBA) which was constituted under the provisions of the Environment
(Protection) Act (EPA) ,1986 . NGRBA has since been dissolved with effect from the 7th October
2016, consequent to constitution of National Council for Rejuvenation, Protection and
Management of River Ganga (referred as National Ganga Council).

The Act envisages five tier structure at national, state and district level to take measures for
prevention, control and abatement of environmental pollution in river Ganga and to ensure
continuous adequate flow of water so as to rejuvenate the river Ganga as below;

· National Ganga Council under chairmanship of Hon’ble Prime Minister of India.

· Empowered Task Force (ETF) on river Ganga under chairmanship of Hon’ble Union Minister of
Water Resources, River Development and Ganga Rejuvenation.

· National Mission for Clean Ganga (NMCG).

· State Ganga Committees and

· District Ganga Committees in every specified district abutting river Ganga and its tributaries
in the states.

IASbaba
Web: http://ilp.iasbaba.com/ Score:
Email: ilp@iasbaba.com 0.00 / 200
Page 62
SET 2 Block 3: Polity
Exam Title :
& Cur...
Email : misrapulkit@yahoo.in
Contact :

NMCG has a two tier management structure and comprises of Governing Council and Executive
Committee. Both of them are headed by Director General, NMCG. Executive Committee has
been authorized to accord approval for all projects up to Rs.1000 crore . Similar to structure at
national level, State Programme Management Groups (SPMGs) acts as implementing arm of
State Ganga Committees. Thus the newly created structure attempts to bring all stakeholders
on one platform to take a holistic approach towards the task of Ganga cleaning and
rejuvenation.

Source: http://pib.nic.in/newsite/PrintRelease.aspx?relid=171223

QUESTION 73.
The tail of a comet is directed away from the sun because

a) as the comet rotates around the sun, the lighter mass of the comet is pushed away due to
the centrifugal force alone
b) as the comet rotates, the lighter mass of the comet is attracted by some star situated in the
direction of its tail
c) the radiation emitted by the sun exerts a radial pressure on the comet throwing its tail
away from the sun
d) the tail of the comet always exists in the same orientation
Correct Answer: C
Your Answer: Unanswered
Explanation

Solution (c)

A comet tail—and coma—are features visible in comets when they are illuminated by the Sun
and may become visible from Earth when a comet passes through the inner Solar System. As a
comet approaches the inner Solar System, solar radiation causes the volatile materials within
the comet to vaporize and stream out of the nucleus, carrying dust away with them. Separate
tails are formed of dust and gases, becoming visible through different phenomena; the dust
reflects sunlight directly and the gases glow from ionisation . Most comets are too faint to be
visible without the aid of a telescope, but a few each decade become bright enough to be visible
to the naked eye.

In the outer Solar System, comets remain frozen and are extremely difficult or impossible to
detect from Earth due to their small size. Statistical detections of inactive comet nuclei in the
Kuiper belt have been reported from the Hubble Space Telescope observations, but these
detections have been questioned, and have not yet been independently confirmed. As a comet
approaches the inner Solar System, solar radiation causes the volatile materials within the
comet to vaporize and stream out of the nucleus, carrying dust away with them. The streams of
dust and gas thus released form a huge, extremely tenuous atmosphere around the comet
called the coma, and the force exerted on the coma by the Sun's radiation pressure and solar
wind cause an enormous tail to form, which points away from the Sun.

Source: http://babapedia2018.iasbaba.com/2017/09/22/22nd-september-binary-
asteroid/

IASbaba
Web: http://ilp.iasbaba.com/ Score:
Email: ilp@iasbaba.com 0.00 / 200
Page 63
SET 2 Block 3: Polity
Exam Title :
& Cur...
Email : misrapulkit@yahoo.in
Contact :
QUESTION 74.
Joint Comprehensive Plan of Action refers to which of the following

a) The Chilcot Report


b) India-Iran Nuclear Deal
c) The Heart of Asia - Istanbul Process
d) None of the above
Correct Answer: D
Your Answer: Unanswered
Explanation

Solution (d)

Joint Comprehensive Plan of Action (JCPOA)

The Joint Comprehensive Plan of Action (JCPOA) known commonly as the Iran deal , is an
international agreement on the nuclear program of Iran reached in Vienna on 14 July 2015
between Iran, the P5+1 (the five permanent members of the United Nations Security
Council—China, France, Russia, United Kingdom, United States—plus Germany) and
the European Union.

Highlights

• Iran had almost 20,000 centrifuges. Under the Joint Comprehensive Plan of Action
(JCPOA), it will be limited to installing no more than 5,060 of the oldest and least efficient
centrifuges at Natanz for 10 years.
• Iran's uranium stockpile will also be reduced by 98% to 300kg (660lbs) for 15 years. It
must also keep its level of enrichment at 3.67%.
• The JCPOA says Iran will not be permitted to build additional heavy-water reactors or
accumulate any excess heavy water for 15 years.
• Inspectors from the IAEA, the global nuclear watchdog, will continuously monitor Iran's
declared nuclear sites and also verify that no fissile material is moved covertly to a secret
location to build a bomb.
• Iran has also agreed to implement the Additional Protocol to their IAEA Safeguards
Agreement, which should allow inspectors to access any site anywhere in the country they
deem suspicious.
• Iran has also agreed not to engage in activities, including research and development,
which could contribute to the development of a nuclear bomb.
• Should Iran violate any aspect of the deal, the UN sanctions will automatically "snap
back" into place for 10 years, with the possibility of a five-year extension.

The Iran Nuclear Deal Explained - https://www.youtube.com/watch?v=03JdbYggUTs

Source: http://www.thehindu.com/news/national/india-walks-a-fine-line-as-us-iran-spar-
over-nuclear-deal/article19733664.ece

QUESTION 75.
Consider the following statements about Advocate on Record

1. Only advocates-on-record can mention petitions for urgent hearing in the Supreme Court

IASbaba
Web: http://ilp.iasbaba.com/ Score:
Email: ilp@iasbaba.com 0.00 / 200
Page 64
SET 2 Block 3: Polity
Exam Title :
& Cur...
Email : misrapulkit@yahoo.in
Contact :

2. No advocate other than an advocate on record can appear and plead in any matter unless he
is instructed by an advocate on record

Select the correct statements

a) 1 Only
b) 2 Only
c) Both 1 and 2
d) Neither 1 nor 2
Correct Answer: C
Your Answer: Unanswered
Explanation

Solution (c)

Advocate on Record is an advocate who is entitled under the Order IV of the Supreme Court
Rules, 2013 (previously Order IV of the Supreme Court of India Rules, 1966), framed under
Article 145 of the Constitution, to act as well as to plead for a party in the Supreme Court of
India. As per the Rules, no advocate other than an advocate on record shall be entitled to file an
appearance or act for a party in the Supreme Court of India. No advocate other than an
advocate on record can appear and plead in any matter unless he is instructed by an advocate
on record.

Only advocates-on-record can now mention petitions for urgent hearing in the Supreme Court
as Chief Justice of India Dipak Misra put an end to the practice of senior advocates queuing up
for it.

Source: http://babapedia2018.iasbaba.com/2017/09/22/22nd-september-advocates-on-
record/

QUESTION 76.
Project Yash Vidya is a

a) bachelor’s degree Programme designed for the Army Personnel


b) project aimed at universalization and qualitative improvement of primary education in the
remote and socio-economically backward villages of Rajasthan
c) program towards enhancing the employability skills of engineering students by building
capacity of engineering education infrastructure
d) initiative that offers young students a skill development course based on the Tizen OS
platform
Correct Answer: A
Your Answer: Unanswered
Explanation

Solution (a)

Project Yash Vidya

• It is a specially designed Academic Programme for the personnel of the Indian Army in
recognition of their selfless service to the nation.

IASbaba
Web: http://ilp.iasbaba.com/ Score:
Email: ilp@iasbaba.com 0.00 / 200
Page 65
SET 2 Block 3: Polity
Exam Title :
& Cur...
Email : misrapulkit@yahoo.in
Contact :
• A Memorandum of Understanding (MOU) was signed between the YCMOU and the Indian
Army on Yash-Vidya

Do You Know?

Mou Between Indian Army and Jamia Millia Islamia University - facilitate admission of defence
personnel in second or third year Graduation/Post Graduation/Doctorate programme through
the medium of distance learning.

Source: http://pib.nic.in/newsite/PrintRelease.aspx?relid=171020

QUESTION 77.
Which of the following committee was set up to study and evaluate the extant public-private
partnership (PPP) model in India?

a) Seechewal Committee
b) Kelkar committee
c) Dharia Committee
d) AC Shah Committee
Correct Answer: B
Your Answer: Unanswered
Explanation

Solution (b)

Kelkar committee to evaluate PPP in India was a committee set up to study and evaluate the
extant public-private partnership (PPP) model in India. The committee was set up by India's
central government and headed by Vijay Kelkar . The committee was set up following 2015
Union budget of India by the then the then finance minister of India Arun Jaitley . It comprised
10 members. The committee submitted its recommendations to Jaitley on 19 November 2015.

Source: http://babapedia2018.iasbaba.com/2017/09/23/23rd-september-public-private-
partnerships/

QUESTION 78.
Why is Bennu the specific target of the OSIRIS- REx mission?

a) Its composition
b) Its size
c) Its proximity to Earth
d) All of the above
Correct Answer: D
Your Answer: Unanswered
Explanation

Solution (d)

IASbaba
Web: http://ilp.iasbaba.com/ Score:
Email: ilp@iasbaba.com 0.00 / 200
Page 66
SET 2 Block 3: Polity
Exam Title :
& Cur...
Email : misrapulkit@yahoo.in
Contact :

Bennu is accessible to us because of its proximity, crossing paths with Earth's orbit every six
years. Its size is also a major factor that was considered; of the 500,000 or so catalogued
asteroids and about 9,000 near-Earth objects, Bennu is one of the most potentially hazardous
known to humanity. It has a 1/2,700 risk of colliding with Earth between the years 2175 and
2196. Finally, Bennu is a B-type asteroid, a rare primitive specimen that might hold organic
molecules. These compounds could be the key to understanding what may have brought life to
Earth at the formation of the solar system.

Source: http://babapedia2018.iasbaba.com/2017/09/25/25th-september-osiris-rex/

QUESTION 79.
What is the longest mountain chain on the planet?

a) The Himalayas
b) The Everglades
c) The mid-ocean ridge
d) None of the above
Correct Answer: C
Your Answer: Unanswered
Explanation

Solution (c)

The longest mountain range on Earth is called the mid-ocean ridge. Spanning 40,389 miles
around the globe, it's truly a global landmark.

About 90 percent of the mid-ocean ridge system is under the ocean. This system of mountains
and valleys criss -crosses the globe, resembling the stitches in a baseball. It's formed by the
movement of the Earth's tectonic plates.

As the great plates push apart, mountains and valleys form along the seafloor as magma rises
up to fill the gaps. As the Earth's crust spreads, new ocean floor is created. This process
literally renews the surface of our planet.

If you look at a map of the world's volcanoes, you'll find that most of them form along the
boundaries of this great system. In fact, the global mid-ocean ridge system forms the largest
single volcanic feature on the Earth. The mid-ocean ridge consists of thousands of individual
volcanoes or volcanic ridge segments which periodically erupt.

The world's longest above-ground mountain range is the Andes ,[ 1][2] about 7,000 km (4,300
mi) long. The range stretches from north to south through seven countries in South America,
along the west coast of the continent: Venezuela, Colombia, Ecuador, Peru, Bolivia, Chile, and
Argentina. Aconcagua is the highest peak, at about 6,962 m (22,841 ft ).

QUESTION 80.
Consider the following statements

IASbaba
Web: http://ilp.iasbaba.com/ Score:
Email: ilp@iasbaba.com 0.00 / 200
Page 67
SET 2 Block 3: Polity
Exam Title :
& Cur...
Email : misrapulkit@yahoo.in
Contact :

1. The 15th (Imperial Service) Cavalry Brigade was a brigade-sized formation that served
alongside British Empire forces in the Sinai and Palestine Campaign, during the First World
War.

2. The Mysore Lancers were in the 15th Imperial Service as the forces sent by the princely
states of Mysore, Jodhpur and Hyderabad.

Select the correct statements

a) 1 Only
b) 2 Only
c) Both 1 and 2
d) Neither 1 nor 2
Correct Answer: C
Your Answer: Unanswered
Explanation

Solution (c)

15th (Imperial Service) Cavalry Brigade

• The 15th (Imperial Service) Cavalry Brigade was a brigade-sized formation that served
alongside British Empire forces in the Sinai and Palestine Campaign, during the First
World War.
• Originally called the Imperial Service Cavalry Brigade it was formed from Imperial
Service Troops provided by the Indian Princely States of Hyderabad, Mysore, Patiala and
Jodhpur, which each provided a regiment of lancers.
• A maximum of three regiments served in the brigade at any one time.
• The states of Bhavnagar, Kashmir, Kathiawar and Idar provided smaller detachments for
the brigade, which was at times reinforced by other British Empire regiments and artillery
batteries when on operations.

Source: http://pib.nic.in/newsite/PrintRelease.aspx?relid=171058

QUESTION 81.
Consider the following statements about Graded Surveillance Measure (GSM) which was in
news recently

1. The GSM is a system designed by SEBI to keep a check on shares which see an abnormal
price rise not commensurate with the financial health or fundamentals.

2. The GSM tags cannot be challenged

Select the correct statements

a) 1 Only
b) 2 Only
c) Both 1 and 2
d) Neither 1 nor 2
Correct Answer: A
Your Answer: Unanswered

IASbaba
Web: http://ilp.iasbaba.com/ Score:
Email: ilp@iasbaba.com 0.00 / 200
Page 68
SET 2 Block 3: Polity
Exam Title :
& Cur...
Email : misrapulkit@yahoo.in
Contact :
Explanation

Solution (a)

Securities and Exchange Board of India ( Sebi ) directed exchanges to suspend trading in 331
listed companies, suspected of being "shell" entities. It directed stock exchanges to place these
companies under the Graded Surveillance Measure (GSM).

The GSM is a system designed by Sebi to keep a check on shares which see an abnormal price
rise not commensurate with the financial health or fundamentals. These companies are often
illiquid, have low market capitalisations and poor fundamentals.

The regulator may suspect that shares of these companies are being bid up and used for money
laundering.

Thus SEBI's intention is to identify and protect investors from dealing in such shares at an early
stage. Such shares are monitored for sudden changes in earnings, book value, fixed assets, net
worth, and price to earnings multiples, among other factors.

Once a company is identified, it goes in to one of the six stages of the GSM, attracting the
corresponding surveillance action. Media reports suggest that over 700 companies have come
under GSM since its introduction in February this year.

There are a total of six stages in GSM where the restrictions on trading in the securities get
progressively higher. In the first stage, securities are placed under the trade-to-trade category,
and a maximum of 5 per cent price movement in shares is allowed.

From the second stage a levy called the Additional Surveillance Deposit (ASD) is applicable for
trading in these shares. The ASD will be retained by the exchange for a period of 5 months. So
the second stage not only involves a 5 per cent price band for shares, it also attracts an ASD of
100 per cent.

In the third stage, buyers are allowed to trade in the security only once a week, i.e every
Monday, apart from paying an ASD of 100 per cent on the traded value, whereas stage four
attracts an ASD of 200 per cent.

Similarly, in the fifth stage, in addition to an ASD of 200 per cent, trading is permitted only on
the first Monday of a month. The sixth stage, where companies are placed, attracts far higher
restrictions.

In this stage trading in these shares are permitted only once a month, that is on the first
Monday of the month without any upward movement in price.

There will be a review process by Sebi twice a year where the shares of companies will be
moved in or out of GSM. Also, a quarterly review of GSM stages will be done where qualified
companies may be moved back from a higher to lower stage.

The companies placed under GSM can challenge Sebi or stock exchanges, the Securities
Appellate Tribunal or the high courts for relief.

Read More - https://www.nseindia.com/invest/content/


FAQs_Graded_Surveillance_Measure.pdf

Source: http://www.thehindu.com/todays-paper/tp-business/all-you-need-to-know-
about-graded-surveillance-measure/article19749506.ece

IASbaba
Web: http://ilp.iasbaba.com/ Score:
Email: ilp@iasbaba.com 0.00 / 200
Page 69
SET 2 Block 3: Polity
Exam Title :
& Cur...
Email : misrapulkit@yahoo.in
Contact :

QUESTION 82.
Identify the 'element'—

1. BARC has recently developed a simple, user friendly, quick and cost effective kit for onsite
determination of this 'element' and to check its water-contamination capacity.

2. The element is widely used in various industries like leather, steel, chrome plating, paint
manufacturing, wood preservation etc.

3. One of its types is toxic and the World Health Organization has classified it as carcinogenic
and has mentioned that it can cause stomach ulcers and cancers, and severe damage to kidneys
and liver.

Choose the correct 'element' being discussed about:

a) Cadmium
b) Iron
c) Chromium
d) Beryllium
Correct Answer: C
Your Answer: Unanswered
Explanation

Solution (c)

Chromium-

· Chromium is widely used in various industries like leather, steel, chrome plating, paint
manufacturing, wood preservation etc.

· Untreated effluents from these industries cause widespread contamination of water as been
reported in several parts of the country.

· Chromium in the environment primarily exists as Trivalent Chromium Cr (III) and Hexavalent
Chromium Cr( VI). The later is toxic and the World Health Organization has classified it as
carcinogenic that also causes stomach ulcers, and cancers and severe damage to kidneys and
liver.

· As per Indian standard IS10500 for drinking water, the maximum permissible concentration of
Cr (VI) in drinking water is 50 microgram per litre . The US Environmental Protection Agency
(EPA) recommends a still lower permissible concentration of 10 microgram per litre .

Detection of Cr (VI) -

At such low levels it is not only technically challenging but also expensive and time consuming
since it involves collection of water samples from affected areas, transportation to laboratory,
storage and finally analysis. The method can be used for limited water samples with errors due
to conversion of Cr (III) to Cr( VI) and vice versa during transport and storage.

BARC-

· BARC has developed a simple, user friendly, quick and cost effective kit for onsite
determination of Cr (VI), which meets IS10500 as well as EPA criterion.

IASbaba
Web: http://ilp.iasbaba.com/ Score:
Email: ilp@iasbaba.com 0.00 / 200
Page 70
SET 2 Block 3: Polity
Exam Title :
& Cur...
Email : misrapulkit@yahoo.in
Contact :

· It provides the much needed solution to measure the level of Chromium contamination in
drinking water and tap water, lakes, rivers as well as ground water.

· Procedure involves adding a specified amount of specific reagents to the water sample and
identifying the developed colour .

Colour chart-

· Colour develops within 5 minutes and the distinction can be made with naked eye.

· For ease of comparison a colour chart is provided with the kit.

· Water samples can be immediately categorized as being safe or toxic for drinking from
Chromium (VI) point of view.

· Kit provides several advantages including onsite detection and instantaneous results,
elimination of use of sophisticated instruments for analysis, low investment on infrastructure
for production of the kit, easy availability of raw materials and very good accuracy for the
intended purpose.

Source: http://babapedia2018.iasbaba.com/2017/09/26/26th-september-chromium-vi/

QUESTION 83.
Recently Oxytocin drug was in news. Consider the following statements

1. It used to induce labour in women

2. The manufacture and sale of the drug without licence is a cognizable and non-bailable
offence under the Drugs and Cosmetics Act

3. It is used as a growth promoter in chickens

Select the correct statements

a) 1 and 2
b) 2 and 3
c) 1 and 3
d) All of the above
Correct Answer: D
Your Answer: Unanswered
Explanation

Solution (d)

The Drugs and Cosmetics Act explains the manner in which Oxytocin can be prescribed and the
detailed records which need to be kept by registered medical practitioners. The records need to
be maintained for three years. The manufacture and sale of the drug without licence is a
cognizable and non-bailable offence under the Drugs and Cosmetics Act.

Despite a ban on the retail sale of hormone drug Oxytocin (that is used to induce labour in
women) for veterinary use, several pharmacies in the State continue to sell it illegally over the

IASbaba
Web: http://ilp.iasbaba.com/ Score:
Email: ilp@iasbaba.com 0.00 / 200
Page 71
SET 2 Block 3: Polity
Exam Title :
& Cur...
Email : misrapulkit@yahoo.in
Contact :
counter to dairy owners and farmers, who use it to boost milk production. In the long run, it
lowers the lifespan of cows and makes them infertile sooner.

Source: http://www.thehindu.com/todays-paper/oxytocin-misuse-fiat-to-state-drug-
controllers/article19754850.ece

QUESTION 84.
Consider the following statements about River Terrapin

1. It is native to India

2. It is extinct in the Wild in India and Bangladesh

Select the correct statements

a) 1 Only
b) 2 Only
c) Both 1 and 2
d) Neither 1 nor 2
Correct Answer: A
Your Answer: Unanswered
Explanation

Solution (a)

The northern river terrapin ( Batagur baska ) is a species of riverine turtle native to Southeast
Asia. It is classified Critically Endangered by the IUCN and considered extinct in much of its
former range.

Considered Endangered in peninsular Malaysia and Indonesia; Critically Endangered in


Bangladesh and India because the subpopulations are all very small and declining. It is Extinct
in the Wild in Thailand (considered CR in OEPP 1997). There are no recent data and it is
therefore presumed to be Extinct in Myanmar, Viet Nam and Singapore.

Native: Bangladesh; Cambodia; India; Indonesia; Malaysia

Regionally extinct: Myanmar; Singapore; Thailand; Viet Nam

Source: http://www.thehindu.com/news/national/other-states/a-fragile-ark-that-
shelters-2626-creatures/article19672202.ece

QUESTION 85.
Vishwajeet Scheme aims

a) To make India a manufacturing hub


b) To make India a defence hub in line with Make in India initiative
c) To put IITs in top league of global rankings
d) To facilitate protection of Patents, Trademark and Designs of innovative and interested
Start Ups.

IASbaba
Web: http://ilp.iasbaba.com/ Score:
Email: ilp@iasbaba.com 0.00 / 200
Page 72
SET 2 Block 3: Polity
Exam Title :
& Cur...
Email : misrapulkit@yahoo.in
Contact :
Correct Answer: C
Your Answer: Unanswered
Explanation

Solution (c)

The scheme entailed the provision of Rs . 1,250 crore to each of the top seven IITs over a period
of five years to upgrade infrastructure, hire foreign faculty, and collaborate with foreign
institutions to break into the top league in global rankings.

Source: http://www.thehindu.com/todays-paper/tp-national/vishwajeet-scheme-a-non-
starter/article19673713.ece

QUESTION 86.
The term Emergency may be defined as a difficult situation arising suddenly and demanding
immediate action by public authorities under power specially granted to them by the
Constitution or otherwise to meet exigencies. Which of the following article/s of Indian
Constitution doesn’t explicitly mention the term ‘Emergency’?

1. Article 352

2. Article 360

3. Article 356

Select the correct code

a) 1 and 2
b) 2 only
c) 3 only
d) 2 and 3
Correct Answer: C
Your Answer: Unanswered
Explanation

Solution (c)

Yes you read it right!

356- Provisions in case of failure of constitutional machinery in State

Article 356 doesn’t mention the word ‘Emergency’

It is called as State Emergency

352. Proclamation of Emergency

360. Provisions as to financial emergency

Note- This is something you should never forget/Revise President’s Rule from Value Add Notes

IASbaba
Web: http://ilp.iasbaba.com/ Score:
Email: ilp@iasbaba.com 0.00 / 200
Page 73
SET 2 Block 3: Polity
Exam Title :
& Cur...
Email : misrapulkit@yahoo.in
Contact :
QUESTION 87.
According to the Constitution of India, which of the following are fundamental for the
government of the country?

a) Fundamental Duties (FDs)


b) Directive Principles of State Policy (DPSP)
c) Fundamental Rights (FRs)
d) Both FR and FD
Correct Answer: B
Your Answer: Unanswered
Explanation

Solution (b)

Directive Principle of State Policy provides guidelines to Central and State government in India,
to be kept in mind while framing laws and policies and are mentioned in Part IV of the
constitution.

The provisions contained in this Part cannot be enforced by any court, but these principles are
fundamental in the governance of the country and it shall be the duty of the State to apply these
principles in making laws.

QUESTION 88.
Consider the following provisions under the Directive Principles of State Policy (DPSP) as
enshrined in the Constitution of India:

1. To promote international peace and amity

2. Provide free and compulsory education for children below 14 years.

3. Promotion of educational and economic interests of the SCs, the STs and the other weaker
sections of the society.

4. Equal justice and free legal aid.

Which of the above are NOT the Gandhian Principles that are reflected in the DPSP?

a) 1 only
b) 1, 2 and 4
c) 2 and 4
d) 1, 2, 3 and 4
Correct Answer: B
Your Answer: Unanswered
Explanation

Solution (b)

Articles 36 to 51 deal with the provisions of the Directive Principles and are broadly classified
into

1. Socialist principles

IASbaba
Web: http://ilp.iasbaba.com/ Score:
Email: ilp@iasbaba.com 0.00 / 200
Page 74
SET 2 Block 3: Polity
Exam Title :
& Cur...
Email : misrapulkit@yahoo.in
Contact :
2. Gandhian principles
3. Liberal intellectual principles

Socialist Principles

1. To secure a social order for the promotion of welfare of the people.

2. To strive to minimise inequalities of income i.e. operation of the economic system does not
result in the concentration of wealth and means of production to the common detriment;

3. ownership and control of the material resources of the community are so distributed as best
to subserve the common good;

4. Equal justice and free legal aid.

5. Ownership and control of material resources of the community shall be so distributed so as to


subserve the common good.

6. Equal pay for equal work.

7. Health and strength of workers, and the tender age of children must not be abused.

8. Right to work, to education and to public assistance in certain cases.

9. Provision of just and humane conditions for work and maternity relief.

10. Participation of workers in the management of the industries.

11. Duty of the State to raise the level of nutrition and the standard of living and to improve
public health.

12. Children are given opportunities and facilities to develop in a healthy manner and in
conditions of freedom and dignity and that childhood and youth are protected against
exploitation and against moral and material abandonment.

Liberal Principles

1. Uniform Civil Code for the citizens.


2. Provide free and compulsory education for children below 14 years.
3. Separation of Judiciary from Executive.
4. To promote international peace and amity.
5. Protection of monuments and places and objects of national importance
6. Protection and improvement of environment and safeguarding of forests and wild life.

The Gandhian Principles

1. Organization of Village Panchayats and to promote cottage industry.


2. Promotion of educational and economic interests of the SCs, the STs and the other weaker
sections of the society.
3. To bring about the prohibition of intoxicating drinks and drugs that are injurious to health.
4. Organization of agriculture and animal husbandry on modern and scientific lines to
prohibit the slaughter of cows, calves and other milch and draught animals.

QUESTION 89.

IASbaba
Web: http://ilp.iasbaba.com/ Score:
Email: ilp@iasbaba.com 0.00 / 200
Page 75
SET 2 Block 3: Polity
Exam Title :
& Cur...
Email : misrapulkit@yahoo.in
Contact :

Which of the following provisions of the Constitution of India have a bearing on Education?

1. Directive Principles of State Policy

2. Rural and Urban Local Bodies

3. Fifth Schedule

4. Sixth Schedule

5. Seventh Schedule

Select the correct answer using the codes given below:

a) 1 & 2 Only
b) 3, 4, & 5 Only
c) 1, 2 & 5 only
d) 1, 2, 3 ,4 & 5
Correct Answer: C
Your Answer: Unanswered
Explanation

Solution (c)

Directive Principles

Article 41. The State shall, within the limits of its economic capacity and development, make
effective provision for securing the right to work, to education and to
public assistance in cases of unemployment, old age, sickness and disablement, and in other
cases of undeserved want.

Article 45: The State shall endeavour to provide early childhood care and education for all
children until they complete the age of six years

Rural & Urban Local Bodies:

Article 350A: It shall be the endeavour of every State and of every local authority within the
State to provide adequate facilities for instruction in the mother-
tongue at the primary stage of education to children belonging to linguistic minority groups;
and the President may issue such directions to any State as
he considers necessary or proper for securing the provision of such facilities.

Seventh Schedule:

Union List, State List and Concurrent List

http://lawmin.nic.in/olwing/coi/coi-english/Const.Pock%202Pg.Rom8Fsss(35).pdf

IASbaba
Web: http://ilp.iasbaba.com/ Score:
Email: ilp@iasbaba.com 0.00 / 200
Page 76
SET 2 Block 3: Polity
Exam Title :
& Cur...
Email : misrapulkit@yahoo.in
Contact :
QUESTION 90.
Consider the following regarding the significance of ‘Fundamental Duties (FDs)’

1. It serves as a reminder to the citizens that while enjoying their rights, they should also be
conscious of duties they owe to their country, their society and fellow citizens

2. They help the courts in determining the constitutional validity of a law.

3. It serves as a warning against the national and antisocial activities

Select the correct statement/s

a) 1 and 3
b) 1 and 2
c) Only 3
d) 1, 2 and 3
Correct Answer: D
Your Answer: Unanswered
Explanation

Solution (d)

Significance of FDs

• It serves as a reminder to the citizens that while enjoying their rights, they should also be
conscious of duties they owe to their country, their society and fellow citizens.
• It serves as a warning against the national and antisocial activities (destroying pub
property/burning national flag)
• Serve as a source of inspiration and promotes discipline and commitment; creates a
feeling that citizens are not mere spectators but active participants in the realization of
national goals.
• They help the courts in determining the constitutional validity of a law.
• They are enforceable by law. Hence, Parliament can provide for the imposition of penalty/
punishment for failure to fulfill any of them.
• Their inclusion helps in strengthening the democracy.

Features of the FDs

1. Some of them are Moral Duties (Ex. Cherishing noble ideals of freedom struggle)

2. Some of them are Civic Duties (Ex. Respecting the institution, National Flag/Anthem)

3. They refer to such values which have been a part of the Indian tradition, mythology,
religions and practices.

4. Essentially contain just a codification of tasks integral to the Indian way of life.

5. FDs are confined to citizens only, do not extend to foreigners (FRs = extend to all à
Citizens + Foreigners)

6. Like DPSPs, FDs are also non-justiciable à Constitution doesn’t provide for the direct
enforcement by the courts.

There is no legal sanction against their violation (however the Parliament is free to enforce
them by suitable legislation)

IASbaba
Web: http://ilp.iasbaba.com/ Score:
Email: ilp@iasbaba.com 0.00 / 200
Page 77
SET 2 Block 3: Polity
Exam Title :
& Cur...
Email : misrapulkit@yahoo.in
Contact :

QUESTION 91.
Consider the following statements regarding the features Jammu and Kashmir Constitution

1. It declares the State of J&K to be an integral part of India

2. It provides for Governor’s rule as appointed by President of India

3. It clarifies that the permanent residents of the state are entitled to all rights guaranteed
under the Constitution of India

Select the correct statements

a) 1 and 2
b) 2 and 3
c) 1 and 3
d) 1, 2 and 3
Correct Answer: D
Your Answer: Unanswered
Explanation

Solution (d)

The Constitution of J&K was adopted on 17 November 1957, and came into force on 26 January
1957. Its salient features (as amended from time to time) are as follows:

· It declares the State of J&K to be an integral part of India.

· It secures justice, liberty, equality and fraternity to the people of the state.

· It says that the State of J&K comprises all the territory that was under the ruler of the state on
15 August 1947. This means that the territory of the state also includes the area which is under
the occupation of Pakistan.

· It provides for Governor’s Rule. Hence, the governor, with the concurrence of the President of
India, can assume to himself all the powers of the state government, except those of the high
court.

· It vests the executive powers of the state in the governor appointed by the president for a
term of five years.

QUESTION 92.
Consider the following and select the correct match:

1. Article 371-A: Special provision for Nagaland

2. Article 371-C: Special provision for Assam

3. Article 371- G: Special provision for Mizoram

Select the correct code

IASbaba
Web: http://ilp.iasbaba.com/ Score:
Email: ilp@iasbaba.com 0.00 / 200
Page 78
SET 2 Block 3: Polity
Exam Title :
& Cur...
Email : misrapulkit@yahoo.in
Contact :

a) 1 only
b) 2 and 3
c) 1 and 3
d) 1, 2 and 3
Correct Answer: C
Your Answer: Unanswered
Explanation

Solution (c)

Articles 371 to 371-J in Part XXI of the constitution contain special provisions for eleven states
viz., Maharashtra, Gujarat, Nagaland, Assam, Manipur, Andhra Pradesh, Sikkim, Mizoram,
Arunachal Pradesh, Goa and Karnataka.

· Article 371-A makes the following special provisions for Nagaland

· Under Article 371-B, the President is empowered to provide for the creation of a committee of
the Assam Legislative Assembly consisting of the members elected from the Tribal Areas of the
state and such other members as he may specify.

· Article 371-C makes the following special provisions for Manipur

· Articles 371-D and 371-E contain the special provisions for Andhra Pradesh or Telangana

· The 36th Constitutional Amendment Act of 1975 made Sikkim a full-fledged state of the Indian
Union. It included a new Article 371-F containing special provisions with respect to Sikkim

· Article 371-G specifies the following special provisions for Mizoram

For details refer to Chapter “Special Provisions for Some States”- Laxmikanth

QUESTION 93.
Which of the followings is/are NOT related to Scheduled Areas/Tribes?

1. Article 244

2. 91 st Constitutional Amendment

3. Article 339

4. Article 332

Select the correct code

a) 1 and 3
b) 1 only
c) 1, 3 and 4
d) 1, 2, 3 and 4
Correct Answer: A
Your Answer: Unanswered
Explanation

IASbaba
Web: http://ilp.iasbaba.com/ Score:
Email: ilp@iasbaba.com 0.00 / 200
Page 79
SET 2 Block 3: Polity
Exam Title :
& Cur...
Email : misrapulkit@yahoo.in
Contact :

Solution (a)

Article 244 in Part X of the Constitution envisages a special system of administration for certain
areas designated as scheduled areas and tribal areas.

The Fifth Schedule of the Constitution deals with the administration and control of scheduled
areas and scheduled tribes in any state except the four states of Assam, Meghalaya, Tripura and
Mizoram. The Sixth Schedule of the Constitution, on the other hand, deals with the
administration of the tribal areas in the four northeastern states of Assam, Meghalaya, Tripura
and Mizoram.

Article 332- Reservation of seats for Scheduled Castes and Scheduled Tribes in the
Legislative Assemblies of the States.

91 st Amendment- Restrict the size of council of ministers to 15% of legislative members & to
strengthen Anti Defection laws

QUESTION 94.
Consider the following

1. International Maritime Organization (IMO) is a specialized agency of the United Nations


responsible for regulating shipping.

2. The World Maritime Day (WMD) is observed every year on September 28 to highlight
importance of shipping safety, maritime security and marine environment and to acknowledge
maritime industry.

Which of the above statements is/are correct?

a) 1 only
b) 2 only
c) Both 1 and 2
d) Neither 1 not 2
Correct Answer: C
Your Answer: Unanswered
Explanation

Solution (c)

IASbaba
Web: http://ilp.iasbaba.com/ Score:
Email: ilp@iasbaba.com 0.00 / 200
Page 80
SET 2 Block 3: Polity
Exam Title :
& Cur...
Email : misrapulkit@yahoo.in
Contact :

The World Maritime Day (WMD) is observed every year on September 28 to highlight
importance of shipping safety, maritime security and marine environment and to acknowledge
maritime industry.

The 2017 theme of the day is ‘Connecting Ships, Ports and People’. It was chosen to
provide an opportunity to focus on many diverse actors involved in shipping and logistics
industry. It aims to focus on helping International Maritime Organization (IMO) member states
to develop and implement maritime strategies to invest in a joined-up, interagency approach
that addresses whole range of issues, including facilitation of maritime transport, increasing
efficiency, navigational safety, protection of marine environment, and maritime security.

IMO is a specialised agency of the United Nations responsible for regulating shipping. It was
established in 1948 in Geneva as ICMO and came into force in 1959. Its headquarters are in
London, United Kingdom.

QUESTION 95.
Recently a new species called ‘Aquatic Rhabdops ’ has been discovered in Western Ghats. It
belongs to:

a) A new species of earthworm


b) A new species of snake
c) A new species of frog
d) A new species of turtle
Correct Answer: B
Your Answer: Unanswered
Explanation

Solution (b)

A new species of non-venomous aquatic snake named Aquatic Rhabdops , which was
misidentified since 1863, was identified and described as new species from northern Western
Ghats in Maharashtra, Goa and North Karnataka. Earlier the new species, Rhabdops aquaticus ,
was considered as variant of Olive Forest Snake, first described in 1863. But new study has
confirmed that Aquatic Rhabdops has different colours and patterns and also vary in other
features of size, shape and structure, and also genetic make-up.

http://www.thehindu.com/sci-tech/energy-and-environment/western-ghats-throw-up-a-new-
snake/article19764405.ece

QUESTION 96.
Consider the following regarding ‘Rabies’

1. Rabies is a vaccine-preventable viral disease


2. Bats, foxes, and skunks are common reservoirs of this disease
3. It affects central nervous system

IASbaba
Web: http://ilp.iasbaba.com/ Score:
Email: ilp@iasbaba.com 0.00 / 200
Page 81
SET 2 Block 3: Polity
Exam Title :
& Cur...
Email : misrapulkit@yahoo.in
Contact :

Which of the given statements is/are correct?

a) 1 and 2
b) 1 only
c) 2 and 3
d) 1, 2 and 3
Correct Answer: D
Your Answer: Unanswered
Explanation

Solution (d)

All the statements are correct

Rabies is a vaccine-preventable viral disease which occurs in more than 150 countries and
territories.

Dogs are the main source of human rabies deaths, contributing up to 99% of all rabies
transmissions to humans.

Rabies is spread through a bite from an infected mammal, usually a dog that has been attacked
by a wild animal. Bats, foxes, and skunks are common reservoirs of this disease.

28 th September- World Rabies Day

The World Rabies Day is observed every year on 28 September across the world to raise
awareness about rabies disease and its preventable measures. The 2017 theme is ‘Rabies:
Zero by 30’. Observance of the day seeks to raise awareness about the impact of rabies on
animals and human beings. It also seeks to share information on how to prevent disease and
highlight progress in defeating this horrifying disease.

QUESTION 97.
Recently GoI has launched ‘PENCIL’ Portal. It is associated with

a) Human Rights
b) Child Labor
c) Women Rights
d) Insurance Sector
Correct Answer: B
Your Answer: Unanswered
Explanation

Solution (b)

IASbaba
Web: http://ilp.iasbaba.com/ Score:
Email: ilp@iasbaba.com 0.00 / 200
Page 82
SET 2 Block 3: Polity
Exam Title :
& Cur...
Email : misrapulkit@yahoo.in
Contact :

The Union Ministry of Labour and Employment launched Platform for Effective Enforcement for
No Child Labour (PENCIL) Portal at National Conference on Child Labour . The PENCIL portal
is an electronic platform that aims at involving Centre, State, District , Governments, civil
society and general public in achieving the target of child labour free society.

http://pib.nic.in/newsite/PrintRelease.aspx?relid=171096

QUESTION 98.
Consider the following w.r.t ‘World Nuclear Energy Industry Status Report 2017’:

1. India is third in the world in the number of nuclear reactors being installed
2. The number of nuclear reactor units under construction is declining globally

Which of the given statements is/are correct?

a) 1 only
b) 2 only
c) Both 1 and 2
d) Neither 1 nor 2
Correct Answer: C
Your Answer: Unanswered
Explanation

Solution (c)

Both the statements are correct

India is third in the world in the number of nuclear reactors being installed, at six, while China
is leading at 20, the World Nuclear Industry Status Report 2017, released this month, shows.
The number of nuclear reactor units under construction is, however, declining globally for the
fourth year in a row, from 68 reactors at the end of 2013 to 53 by mid-2017, the report says.

http://www.thehindu.com/news/national/india-third-in-nuclear-power-installations-study/
article19744091.ece

QUESTION 99.
PRASAD Scheme is undertaken by which ministry?

a) Ministry of Rural Development


b) Ministry of Urban Development
c) Ministry of Tourism
d) Ministry of Minority Affairs
Correct Answer: C
Your Answer: Unanswered
Explanation

Solution (c)

IASbaba
Web: http://ilp.iasbaba.com/ Score:
Email: ilp@iasbaba.com 0.00 / 200
Page 83
SET 2 Block 3: Polity
Exam Title :
& Cur...
Email : misrapulkit@yahoo.in
Contact :

Read this link in detail

http://pib.nic.in/newsite/PrintRelease.aspx

QUESTION 100.
Consider the following regarding recently notified ‘Wetlands (Conservation and Management)
Rules, 2017’

1. It stipulates setting up of State Wetland Authority in each State/UTs headed by State’s


environment minister and include range of government officials.
2. The rules stipulates for setting up of National Wetland Committee, headed by Ministry of
Environment and Forest Secretary, to monitor implementation of these rules and oversee
work carried out by States.
3. The new rules has replaced the 2010 version of the rules

Which of the given statements is/are correct?

a) 1 and 2
b) 3 only
c) 2 and 3
d) 1, 2 and 3

Correct Answer: D
Your Answer: Unanswered
Explanation

Solution (d)

All the statements are correct

http://www.livemint.com/Politics/y6Tr3tkrr3q28AmGKaBFII/Environment-ministry-notifies-new-
wetland-rules.html

IASbaba
Web: http://ilp.iasbaba.com/ Score:
Email: ilp@iasbaba.com 0.00 / 200
Page 84
SET 2 Block 3: Polity
Exam Title :
& Cur...
Email : misrapulkit@yahoo.in
Contact :
Review in

IASbaba
Web: http://ilp.iasbaba.com/ Score:
Email: ilp@iasbaba.com 0.00 / 200
Page 85

You might also like